Вы находитесь на странице: 1из 71

MCCQE 1, November 2003 1. For nocturnal enuresis how long you will give desmopressin? A. 1-2 months B.

. 2-3 months C. 3-6 months D. 6-9 months E. 1-2 weeks 2. A lady is allergic to lidocaine. She is going to have a vulval biopsy. What will you give? A. topical preparation of prilocaine and lidocaine B. prilocaie infiltration C. infiltration of Bupivacaine D. no anesthesia 3. which of the following is not a complication of abruption placenta? A. pulmonary embolism B. vaginal bleeding leading to shock C. renal failure D. extravasation of blood into myometrium E. DIC 4. A child 3years came to ER with a history of bleeding per rectum since 2 hours. She was healthy before that. She bled 300ml of blood. After coming the ER she again had a bout of bleeding of about 200ml. PE- abdomen soft, no mass palpable. What is the most appropriate investigation? A .ultra sound B lower GI endo scopy C. Technetium 99 scan D.Ba enema 5. A lady one year after delivery presents with amenorrhea even though She chose not to breast feed her baby . she had PPH and curetting done during delivery. You gave her estrogenprogesterone. But no withdrawal bleeding. what is your diagnosis? A. Sheehan syndrome B. Asherman syndrome C. premature ovarian failure 6. similar scenario. But here she had PPh but curetting not done. She had withdrawal bleeding when estrogen-progesterone given. What is your diagnosis? A. sheehan syndrome. B. premature ovarian failure C. asherman syndrome. 7. an epileptic man getting phenytoin 400mg. epilepsy is not controlled. What are you going to do? Increase the dose. 8.what is the prevalence of anxiety disorders in children? A. 2% B.5% C.10% D.15% 9.picture of rectal prolapse in a child. Which of the following is true about cystic fibrosis? a. affected parents common

b. parents may have subclinical disease c. antenatal diagnosis may be possible. 10. question on nerve injury, a man cannot touch his index finger with thumb. Sensations over palm normal. No other abnormality. Which nerve injury? A. ulnar nerve B. median nerve C. radial nerve D. Anterior interosseous nerve E. axillary nerve. 11. Which of the following is unlikely to be due to child abuse. A. fouchette tear B.Hymen tear C. Condyloma D. Chalmydia infection E. Vulval hematoma 12. 2 year old child with fever, continuous tonic clonic seizure for 15 minutes what will you give? a. Iv lorazepam b. IV phenobarbitone c. paraldehyde d.ethosuccimide 13. 14 year old boy ,height 5th percentile weight 50th percentile bone age delayed what is the first investigation a. growth hormone assay b. karyotyping c. testosterone 14.parents brought a 7 year old boy to you. His height 5th percentile, he is doing good at school. Parents are of normal height. What inv. Will you do first. a. bone age b. skeletal survey c. karyotyping d. hormonal assay 15. which of the following is most likely to be ectopic pregnancy? a. 7 weeks amenorrhea with vaginal spotting b.7 weeks amenorrhea with IUD c.12 weeks amenorrhea with right lower quadrant pain d.5 weeks amenorrhea with menstrual like cramps. 16. which is the most common cancer in Canada in men(excluding skin cancer) a. lung b.prostate c. colon d.liver e. brain 17. which of the following first appears in ARDS? a. low PO2

b. High PCO2 with Low po2 18. Which of the following is not seen in AIDS related complex? a. lymphadenopathy b. cardiomegaly c. splenomegaly 19. a lady 65 years taking beta blocker for hypertension. She has decreased sleep and the doctor prescribed imipramine. She complains of fatigue. Physical exam normal. What is the most likely cause? a.chronic fatigue syndrome b. due to the effect of drugs. c. hypothyroidism 20.tricyclic overdose. Patient develops arrhythmia. What will you give sodium bicarbonate I/V 21.tricyclic overdose. How will you monitor. ECG 22.a patient with heart rate186 with wide QRS complex. how will you differentiate SVT from ventricular arrhythmia? a. fusion beat b. capture beat c. delta wave. 23. same question. Options different. a. fusion beat b. capture beat c. irregular rhythm. 24. how will you differentiate maxillary fracture from zygomatuc fracture? a. malocclusion b. diplopia 25.a lady with 6 months of amenorrhea, pregnancy test positive. She felt fetal movements 2 months back. PE- fundal height-27 cm. what is the approximate gestational age type one answer 26.patient with chronic AF on long term warfarin treatment. PT- 5.9 what will you do? a. cryoprecipitate b. FFP c. Platelet transfusion d. Vitamin K injection 27. a patient known alcoholic, came to ER with bleeding esophageal varices. Patient is hemodynamically stabilized. What is your next most appropriate management? a.IV octreotide b. gastroscopy. c. Senstaken tube

28. a child is brought to the ER by parents with history of diarrhea and vomiting for 3 days. Last episode of vomiting was 12 hours ago. Loose stools 4 times in last 24 hours. PE- pulse-100/mt bp- 100/60 oral mucosa mildly dry and eyes mildly sunken. No other abnormalities. What is the management? He was tolerating oral clear fluids.(part B Q)- choose 2 a.oral electrolye solution b. oral clear fluids c. admit d. send him home e. IV normal saline f. D5W 29. a child has ingested cleaning liquid with lye. Which of the following is not done. a. iv fluids b. NG tube c. chest X ray d. endoscopy e. ETT 30. an 85 year old man living in a nursing home complaints of obstipation for the last 3 days. He was apparently healthy before that. What is the most common cause? a. cancer colon b. fecal impaction c. 31.a man conducted a cohort study about social support on health out come. Here social support is a. determinant b. commodity c. confounding factor. d. 32.an asymptomatic lady, pap smear negative, Chlamydia and HPV negative. On examination cervix shows pink squamous epithelium and inner to that red columnar epithelium around the os . what is your further action? a. pap smear after 3 months. b.Laser coagulation c. Do nothing d. Biopsy 33. a man complaints of thickening and .(suggestive of fungal infection) of toe nails.. How will you treat a. oral terbinafine for 3 months b. oral itraconazole for 12 months c. oral grieseofulvin for 9 months d. topical miconazole for 3 months 34. a lady complaints of pain in the right upper quadrant for the last 24 hours. Tc- 13000, amylase 800, Alk.po4ase 160 sgpt 56 sgot60 s.bilirubin normal what is your diagnosis a. acute cholecystitis b. acute pancreatitis c. acute hepatitis d. choledocholithiasis 35. spontaneous rupture of extensor tendon of thumb occurs in a. rheumatoid arthritis

b. de quervains disease 36. 28 year old lady has weight gain ,fatigue etc.(features suggestive of hypothyroidism )for the last 2 years. On examination there is irregular rubbery enlargement of thyroid. TSH is 9.1 what is your next step? a. give her eltroxin b. do radio active iodine uptake scan c. FNAC d. do T4 e. USS 37. an aboriginal man living on reserve goes to a physician. Who is going to pay for his visit? a. federal govt b. provincial govt c. some aboriginal organization(forgot the name) 38. which of the following is true about Canada health act? a. health insurance is Potable to all provinces and regions b. accessible to all. c. when goes to another province the host province temporarily take care of the cost. d. 39. which of the following organization is for the best interest of physicians? a. Canadian medical association. b. MCC c. 40. a man is in the ER with fracture of femur and comminuted fracture of tibia which is reduced and taken care of. Which all complications can occur? Part B Q a. fat embolism b. compartmental syndrome c. atrophy of quadriceps d. knee joint stiffness e. decubitus ulcer 41.what is the standard treatment in tourette syndrome? a. haloperidol b. family therapy c. individual psychotherapy d. behavior therapy 42. a pregnant lady is going to a vacation in ? african country. Which of the following is not true? a. hepatitis A vaccine can be give b. hepatitis B vaccine can be given c. chloroquine prophylaxis is contra indicated 43. which of the following is not a feature of vaginal foreign body a. dyspareunia b. foul smelling vaginal discharge c.fever d. vaginal bleeding e.abdominal pain 44. a pregnant lady at 37 weeks date confirmed by first trimester scan, fundal height 31 cm. USS done shows decreased liquor and estimated fetal weight 2000 gram. What is your next action. a. BPP twice per week b. repeat scan after 24 hours c. admit her and plan for induction of labor

d. send her home. E contraction stress test ( I am not sure whether this option was there or not) 45. a pregnant lady at 22 weeks, absence of fetal movements. uss- no fetal heart activity. She wants termination. What will you do? a. intra amniotic injection of oxytocin b. CS c. iv oxytocin d. vaginal misoprostol 46. a patient with generalized lymphadenopathy and liver and spleen enlargement. blood values given. Diagnosis is CLL. What is your next action? a. do nothing b. lymph node biopsy c. spleen biopsy d. Philadelphia chromosome. e. CT 47. a man in a battery factory complaints of abdominal cramps , constipation. He does not use the hand protection and does not wash hands properly. What will you next? a. blood lead level. b. peripheral smear c. 48. a 3 year old child complaints of pica, abdominal cramps and constipation. He is pale. What is your diagnosis? a. Iron deficiency anemia b. lead poisoning c. leukemia 49. dacryocystitis- mangement. Part BQ 50. an elderly man complaints of obstipation for 2 days. Plain x-ray abdomen shows , bend shaped shadow which is pointing towards right upper quadrant. Diagnosisa. sigmoid volvulus b. ceacal voulvulus. 51. a diabetic old lady BP- 200/106 lying and 160/86 upright. How will you treat? a. ACE inhibitor b. beta blocker c. thiazide 52. a man with pain in the right upper quadrant, fever. Sgpt 1000 s.bilrubin increased. Diagnosis. a. acute hepatitis b. acute cholecystits c. acute pancreatitis. 53. another similar question.- probable diagnosis- acute cholecystitis. 54. newborn baby bilious vomiting . type one diagnosis. 55. which of the following is a criteria in biophysical profile? a. fetal flexion movement of hands and has to touch chest. b. fetal breathing movements. 56.post partum lady. 2 weeks after delivery crying excessively, anxious ,irritable and sad .Which of the following is most unlikely a. suicidal thoughts b. infanticidal thoughts

c. previous history of phobia and anxiety d. most likely to have underlying mental disorder. e. Ambivalent role of mother (dont remember the exact sentence) 57. a boy fell from a tree. Complaints of pain in the lumbar region and Foleys inserted. Shows hematuria. What is the next most appropriate investigation. a. cystoscopy b. CT scan. c. USS d. 58. a man had injury on his leg. while working in a factory something sharp fell on his leg. Now he presents with pain and swelling lower leg. On examination lower leg and foot swollen and veins dilated. When you press femoral artery radial pulse diminishes in volume. What is the diagnosis. a. AV fistula b. 59. a man complaints of pain and swelling in the popliteal fossa. Pulsating mass seen. What is the investigation of choice. a. venous Doppler b. Ct scan c. atreriography d. venography. 60. 7year old child with Legg-Calve-Perthes Disease.which of the following is most characteristic? a. limitation of internal rotation and abduction of thigh b. apparent shortening of leg. 61. a lady with recurrent attacks of facial rashes, lip swelling. which of the following is true. a. chronic angioedema is not serious. a. respiratory distress can occur. 62. a child with infected lesion on left fore arm , edematous hot and red streaks seen. Which of the following organism is most likely a. hemolytic streptococci b. 63. a lady developed cough and breathing difficulty while working in the office. She found a mould on the wet damaged wall. She thinks that is the causative problem as many other staff also developed the same symptoms. Who can provide most reliable information on that a. her family physician b. occupational therapist c. microbiologist d. building engineer 64. a man had his lung functions tested as part of the occupational surveillance. Fvc is reduced. It is an example of a. disability b. handicap c. impairment. 65. picture of face with suture above eyebrow. Surrounding area red swollen. Dog bite wound. Which of the following is to be done? a. suturing of dog bite wound is not recommended. b. Remove the sutures immediately c. oral antibiotics d. iv antibiotics

66. a baby with birth wt.3.5kg now 4 weeks weighs 3.6 kg. Mother worried not drinking enough milk. What will you advise? a) Tell her that nothing to worry as some kids dont gain much weight in the first month b) Start formula feeding c) Investigate the kid d) Calculate required calories and improve mothers diet e) Refer her to the midwife in the lactation clinic 67. 31 year old lady with a 6 month old baby. She is breast feeding the baby. She attained menarche at the age of 14 and was on BCP for 17 years. Her mother died of breast cancer at the age of 65. she is afraid that she will develop breast cancer. which of the following is the most important risk factor? a. first child birth after 30 years b. mother died of breast cancer at the age os 65 c. BCP for 17 years d. breast fed the baby. e. menarche at 14 years 68. post menopausal .smoker. to prevent osteoporosis a. bisphosphonate 69. In the ER you are planning to do a new trial of a new treatment on patients with retrosternal chest pain. Which of the following is true a) Such studies should not be done on emergency patient as they need immediate management b) They are not having full capacity to give consent d. take consent from SDM. 70. A lady for laparoscopic tubal ligation on the way to theater asking you for removal of a mole on her forearm while she is under GA. She is already on premedication. What you will tell her? a) You cannot do the procedure, as consent should be taken before coming to theatre before giving premedication. b) Agree to do the procedure 71. A lady posted for hysterectomy. While counseling her what will you tell her regarding the risk of thromboembolism a) Risk is highest after 72 hours b) Thromboembolism most commonly originate from pelvic veins 72. which of the following is most suggestive of stress incontinence. a. loss of urine with increase in intra abdominal pressure without detrusor contraction b. loss of urine associated with detrusor contraction c. loss of urine associated with increased intra abdominal pressure and detrussor contraction. 73. amniocentesis and chromosomal analysis shows XYY. Which of the following is most likely seen in XYY syndrome? a. mental retardation b. dysmorphic features c. aggressive behaviour 74.autism a. high incidence is seen in siblings b. autosomal recessive c. separation anxiety when send with strangers 75. A 35- year old male, one of your patientss for four years, has contacted you and asked you to sign a form for him that he was sick last week. He has to present the form to his employer and

mention that during this time he had flu and it was not severe enough to make him come to you and he did not want to abuse the health system. You believe your patient. what will be your action? A. Tell him that this is illegal and you can not sign the form B. Sign the form but indicate that you did not see the patient C. Sign the form because you know your patient very well D. Call his employer and explain what was the situation 76. A 58-year old patient was admitted to the hospital for appendicectomy. During the surgery his condition deteriorated and he passed into coma. You started tube feeding. The patient has no family member or friends around and no advanced directive. When can you stop the tube feeding? A. You can not stop it except by the court order B. You stop it according to the need of his bed for another patient C. appoint legal guardian to take decision D. You can stop it according to the opinion of another physician E. You consult CEO of the hospital to take a legal action. 77. quality of care a. chief of staff 78. 40-year old patient of yours comes to you and tells you that he is separated from his wife and his wife filed a case that he is sexually abusing their child. He was found guilty and was in jail and just released. He said he has got gun and immediately after leaving the hospital he is going to their house to shoot his wife , he knows the time when his wife is coming back from office What will you do? A. Inform the police B. Inform the wife C. You dont have to breech the confidentiality of your patient D. persuade him not to kill his wife E. asks him to give the gun to you. 79. Which of the following true about advanced directive? A. living will is the most valuable (dont remember the exact word) of all advanced directives B. written advanced directives only accepted by court of law. C. cannot be made when patient is healthy 80. SDM should take decision based on A. wishes expressed by the person when healthy. B. what the SDM thinks right based on medical condition and cost of health care C. apparent wishes of the patient if he is faced to such a situation. 81. Which of the following a cause of contagious dementia. a. Alzheimers B. picks disease c. Creutzfeldt-Jakob Disease D. 82. a young lady with complaints of anorgasmia what will you ask in the history a. orgasm with masturbation 83. which of the following is least suggestive of somatoform pain disorder a. pain leading to impaired function b. multiple somatic complaints

c. involuntary 84. rheumatoid arthritis patient on gold and NSAID . complaints of heavy protienuria and facial edema. What was your action a. stop gold b. stop NSAID c. stop both d. give steroids. 85. ECG showing RBBB and AF. In which of the following conditions does it occur? a. VSD b. Mitral stenosis c. pericarditis d. myocardial infarction 86. picture of a child with wasted muscles and protuberant abdomen. What investigation you will do a. intestinal biopsy b. USS c. CT scan d. e. 87. a lady is brought to the ER by her husband with history of injury on the shoulder. On examination, old healed bruises are seen on the other arm. What will you ask in the history. a. domestic violence 88. which of the following is true about domestic violence a. stopping alcoholism stops violence b. during pregnancy violence increases 89. another Q about domestic violence. Which is true. a. man seeks forgiveness during the period of reconsiliation b. always there is a brief period between escalation of tension and explosion c. women does not believe that during the period of escscalation she cannot do anything to prevent that. 90. obese child with which of the following factor needs investigation? a. both parents obese b. short stature c. 91. A young man is brought to the ER after a MVA. It was a head on collision between a small car and his truck. The old lady driving the car died on the spot. This young man has a few bruises on his chest and abdomen. He is otherwise stable and seems to have no serious injuries. In the mean time police arrives and ask you to with draw some blood for the blood alcohol levels. What you will do? A. Ignore the police B. Order the nurse to withdraw blood and label it and send it to the laboratory C. Tell the police that you need a court order e. Ask the person for consent to take blood 92. Now you have ordered the laboratory for the blood alcohol level. What will you tell the police? a. Tell the police that this is confidential information and you cannot hand it over to them. b. As soon as you receive the results of the investigations, hand them over to the police c. ell the police that they have to bring the court order

d. ask the police to give a written request. 93. a person has alcoholic cirrhosis. He is not willing to quit drinking. He wants liver transplantation . but you cannot fulfill his wish because a. there are other patients in the list who have more serious condition b. b. 94. A patient has undergone Laparotomy for appendicitis and you have left a piece of gauze into her abdomen that is discovered on USG. What will be your action? A. Tell her the truth about the gauze. B. Dont tell her anything. C. Do the surgery without telling her. And tell her alter if you find the gauze. D. 95.a5 year old girl is diagnosed to have inoperable brain tumor. Parents request you not to tell her. The next day morning the girl asks you about her disease. What will be your action a. tell her everything b. do according to parents wish c. involve ethicist of the hospital d. 96. 5 year old boy is met with a MVA. He has rupture spleen. Pr 140, bp 80 systolic. He needs blood transfusion. His parents are active Jehovah witness .he has no belief but reluctant to go against parents wish. What is your action a. give blood products only, not whole blood b. dont give blood c. get his consent d. get a court order of the hospital and give blood 97. an old couple had a MVA. Husband died at the spot . wife needs immediate surgery. on the way to theater nurse found a Jehovah witness card in his coat which is neither signed nor witnessed. What will be your action 98. a. give blood transfusion c. dont give blood transfusion 99. which of the following is the duty of coroner a. find out the cause of death and identity of deceased b. assist police in investigation c. find out whether negligence was there. 100. you agree to provide help for a research . you are asked to give names of patients with osteoporosis who come to your clinic. Which of the following is true. a. you have to take consent from those patients to participate in the study b. you give the records of the patients to the researcher c. you give the name of the patients to the researcher d. you get permission from the patient to give their name to the researcher. 101. a lady had laparoscopic tubal ligation. Now has lower abdominal pain . what is the reason. a. acute salpingitis 102. which of the following is suggestive of cardiogenic shock a. low BP, Low CO, raised JVP, increased PR. 103. a patient has CVP 16 , Co decreased, PR increased . diagnosis

a. cardiogenic chock 104. picture- peripheral smear diagnosis. Megaloblastic anemia Iron deficiency 105. a pregnant lady had routine blood work at 13 weeks GTT 5.7 ,VDRL negative, HBS ag negative, Hb- 13. Rubella titer negative. what will you advice a. start iron now b. start folic acid now c. give rubella vaccine now. 106. which of the following is true about huntingtons chorea a. AR b. Both psychological and neurological features are required for diagnosis. c. Either one can be seen alone early I the course of disease. 107.which of the following feature distinguishes maxillary from zygomatic fracture a. diplopia b. malocclusion 108. which of the following is true. Cluster head ache always bilater and develops gradually Migraine always unilateral Migraine commonly associated with nausea, vomiting,photophobia, Common head ache always unilateral 109. picture of lady with prominent and tortuous temporal artery. This condition is associated with a. unilateral headache b. jaw claudication 110. OCD patients seek medical attention after 5-10 years of disease onset. Because a. they think their feeling is normal b. they are afraid to see doctors. c. fear of social isolation d. they feel shame of their obsessions and compulsions 111. a 25 year old man complaints of arthritis, conjunctivitis and urethritis. Which of the following seen with this a. mouth ulcers b. skin rashes 112. 18 month old child does not speak a word. He seems to understand everything his mother says to him. He waves bye bye when she leaves. Started cruising at 12 months and walking at 15 months. What will be your action a. developmental assessment b. reassure and regular follow up. 113. which of the following is not a dysmorphic disorder (Not sure) a. Williams syndrome b. prader villi c. down syndrome d. angel man syndrome.

114. a man while lifting a heavy weight developed sudden pain in back radiating to leg. Involvement of L5 root. Which of the following is true? A. inability to dorsiflex big toe b. heel walking is not possible c. toe walking is not possible. 115. 15 year old athletic girl pain on the knee A. Osgood schlatter disease 116. 28 year old man , painless scrotal mass , tans illumination negative. What is the most appropriate investigation\ a. needle aspiration b. open biopsy c. USS 117. child 2years had UTI. VCUG shows grade 2 reflux. Treatment a. continuous prophylactic antibiotics 118. a child with chronic renal failure, with posterior urethral valve. He is prepared for a ventilation tube insertion. Nil orally for last 12 hours. Throughout the procedure 70 ml /hour ringer lactate given. s.Na. is 150. what is the reason for hypernatremia. a. decreased concentrating capacity of kidney b. excess administration of IV fluids. c. insensible water loss 119 a child with green stick fracture with 10 degree angulation. Treatment. A. POP cast b. correct angulation to zero and put cast c. open reduction d. splint 120.Pt post elective surgery. He is breathing spontaneously at 32 / min. He is extubated in the recovery room. Then he develops dyspnea; crackles and a few wheezes noted at the right base. O2 Sat 89% on O2 28% concentration. What is the cause of his condition? A. Anaphylaxis A. Aspiration B. Spontaneous pneumothorax PE Part B 120. nocturnal enuresis- 1. what will you ask in the history 2. what inv. 3. what advice 121. a 16 y. o. girl comes to the office for OCPs. She started a sexual relationship with her boyfriend 6 months ago. She is in good health, but she smokes 10 cigarettes /day. Q1: what initial investigations would you do? a. ALT b. AST Cervical swab for Chlamydia c. Cervical swab for GC d. Pap smear Pt decides to take OCPs and asks for Rx. Q2: What will you do? a. Give her Rx for OCPs b. Call her parents for consent to give her OCPs c. Recommend that she use condoms all the time

d. e. f. g.

Recommend STD counselling Recommend Pap testing Do not give her Rx for OCPs and ask another colleague to write her Rx? Recommend that she stop smoking

Q3 what is your next actionGive her prescription. 122.pneumothorax. 123. Police was called to the mall by the owners, at 1600 hrs, b/c a man tried to steal. . They found him (pt) "euphoric"and shouting at the bypassers. He kicked a trash can and was looking for empty bottles and cans (for alcohol?). Pt was brought to ER at 1800 hrs. The police want to charge him, but first they want the pt to be cleared medically. Pt said that he only needed some money to buy one thing. (drugs? Alcohol?)In ER pt is sleepy and uncooperative, but with the help of police and staff, you manage to assess him. Pt has slurred speech, ataxia, sweating. BP 170/110, P 120, T 38,1. Q1: Type 3 DDx: Q2 What investigations (if any) would you do? 124. 14 y.o. girl, pale, SOB, intolerant to exercise. Mom says she eats only junk food. Girl also c/o dysmenorrhea and moderately heavy menses. Q1: Type DDx (3) Q2 Your initial mgmt? 125.68 y.o. male comes to your office b/c he &his wife are concerned he had 4 episodes of fainting over the past 1 month. The episodes occurred while pt was sitting on the sofa & watching TV or while sitting at the table. Pt started feeling dizziness & tinnitus, then he lost consciousness for 5-10 seconds. Then c/o blurred vision & slight confusion also for a short period of time. His wife noted jerking movements of the arms. Pt remembered that he fainted. Q1: Type 3possible syndromes. Q2 On P/E, pt is afebrile, BP 140/85 sitting and 135/80 standing. Cardiac & neurologic exams are normal.Pt takes 25 mg HTCZ for many years, without any side effects. At a recent check up K+ was normal. Q2: type 3 possible causes. Q3: What single further investigation would you do?select 2 126. Parents are concerned that their 7 y. o. son has decreased concentration & poor language skills, difficulty spelling & reading. He can sit through the class & is able to do finish his homework. He has been called to the principle's office for a few times b/c he became very aggressive in the schoolyard. Q1: What is the most likely Dx? Type one Dx Learning disorder of reading 127. A mother calls you from her cottage b/c her son woke up that morning with a dead bat beside him. Her husband picked up the bat with a gloved hand & burried it near the cottage. The local animal control officer says that there are brown and silver bat species in that area. Select up to 3: a. Give antidiploid cell rabies vaccine b. Give antidiploid cell rabies Ig c. Notify Public Health d. Arrange appt to see if child has any puncture wounds or scratch marks e. Ask mom to check if child has any puncture wounds or scratch marks f. Call animal control officer to locate the burried bat

128 You are a new doc in your community. A 55 y.o. male c/o chest pain x 1 yr. CP occurs with exertion, no pain at rest. He has seen other doctors, but wants your opinion. Q1: What is your working Dx? Select one. a. Angina b. Unstable angina c. Q2 what investigation 129. (part A) All are associated with sleep apnea except: a. Obesity b. DM c. HTN 130. 60 y.o. male with BPH. Biopsy shows prostate CA, stage A1. Tx? a. Radical prostatectomy b. Radical prostatectomy and radiotherapy c. Hormonal therapy d. Do nothing 131.which of the following is true about opioid withdrawal? A. most often life threatening b. does not need methadone always\ 132. G6P5 term , in active labour cervix 6 cm dilated. BOM bulging, presenting part not felt . what is your next action. a. prepare for CS b. amniotomy c. do USS in laor room d. IV oxyticin 134. XYY male .which is true a. dysmorphic features b. mental retardation c. behavioural problems

pictures: 2 ecgs: atrial fibrillation+ Lbbb ? Acute miocardial infarct? -pictures: keratoacantoma lower lip (old man) -caryotip: X0 (Turner) picture: girl (Turner syndrome) Ask about, EXCEPt: poor school performance -Thyroid duct cyst (pict) -basal temperature chart (woman); anovulatory profile -rectal prolapse 5 years old girl: "ask sweat chloride test" (YOU NEED TO HAVE IN YOUR MIND THE DIAGNOSIS OF CYSTIC FIBROSIS!!) -picture: oval lesion on the back: P. rosea -picture: man (tinea corporis) ASKED: treatment for this condition -picture: X ray hand: diagnosis: CREST (in scleroderma) Questions: -best prognosis: (thyroid cancer) (follicular, papillar, medular).... -treatment onicomycosis -baby of a intercousins marriage: most likely condition in the baby? (malrotation of intestine, pyloric stenosis, annular pancreas, esophageal atresia) -kid bitten by pet (dog) what to do? kill the dog, vaccine the kid, observe for 10 days (confine the dog) -2 questions about malaria therapy: "what canadians take when they go to a malaria area WITHOUT resistance?" and: "African canadian wuth diagnosis of Falciparum, after 2 years in canada relapses the disease.. causes: mistake in the diagnosis> release of liver schizontes? drug resistance? not compliance of treatment? -fat man diabetic shows hypertension at medical office, which cames back to normality after 10 min. what is the cause/management? "this happens in the majority of the patients/ better make measurements at home: this would help in the treatment/ tecnical problem/ prescribe drugs... -major cause of rupture of extensor policis longus? (Dupuyatren, carp syndrome,lunate fracture,RA,scleroderma -what hypomagnesia doesn`t cause?? -Optic neuritis without pupilar envolvement? (DM,arteritis, tumor,etc) -What the governement puts on canadian water to purify it?? -what doesn`t cause occupational asthma?? -kid with abdominal pain+headache+vomits+following sleep (cerebellar tumor? migraine? complex partial seizure? ...) -bacteria found in raw eggs+ raw meat? -manag. in ASCUS -atypias(found in papanicolau) repeat in 4 months? colposcopy+biopsy?? biopsy only? colposcopy only?? I`ll post more later!!!

EE MAY 2004 1. allergy desensitization? 2. persons walk two blocks then rest ? 3. undercooked food and eggs caused by? 4. stree incontinence? 5. in pda? 6. pts has non biolous vomitting? 7. spermatic cord torsion? 8. a girl is 15 year fully sexually active want to use iud? 9. where one lady used IUD? 10. Bicor uterus has one women? 11. which throid tumor has good prognosis? 12. varcilla? 13. toxic shock syndrome? 14. pts has heavy bleeding with uterus emplty? 15. acynotic heart disease? 16. Schizophrenia-imaging changes:prefrontal cortex 17. 10 day newborn jaundice,treatment:stop brestfeeding,phototherapy 18. ac appendicitis missdiagn. ac mesenteric lymphadenitis 19. Man 64 year with hoarseness:laryngoscopy 20. syncope in young man:vasovagal 21. Cause of ac cholecystitis:obstr by stone 22. intest obstr,except:guarding 23. diabetic pregnant; control of glycemia during pregn 24. man with knife(!) in thigh; press fem art 25. esoph atresia:no air shadow 26. clue cells:vaginal secr,KOH,cover 27. neonatal sepsis:skin eruptions?hypothermia? 28. man with dark skin,hepatomegalia:hemochromatosis 29. ophtalmoplegia with pupillary sparing:diabetes 30. hemophilia A,carrier 1/4 31. Huntington:50% 32. baby with cleft palate:feeding problem 33. peptic ulcer:penetration 34. Hirschprung d:from birth 35. Hirschprung dg: X Ray 36. an infant with running nose plus wheeze? 37. disseminted intravascular bleeding? 38. a patient have gangrenous foot and doctor want to amputate what decision? 39. mania treated with? 40. gender identity? 41. rectal bleeding in infant? 42. in picture of an infant in which when mother change diaper what will doctor do? 43. patient has jaundice plus Alanine increased plus aspirate increased what diagnosi? 44. acute appendicitis is misdiagnosed with? 45. common symptom of sheehan syndrome? 46. 1q's about psa 47. hiv result what will u tell to pt 48. retinal detachment 49. oribtal sweeling = oribtal cellulitis

50. hearing test = wax 51. psa 52. +ve node in stage 1a prostate ca what to do 53. 15 % burn child with stridor 54. cleft palate what will be the problems in incoming years 55. consent for surgery what u will not mention 56. q's about case control + cohort study 57. psa livel increase in 58. precocous puperty what is of the 5 answers can be dx with precocus puperty 59. child development 60. bactrial vaginosis 61. in canada all regarding mortality is unequal except 62. q's about primary health care 63. elderly 2 yrs cough + chr b/l basal field hassiness = histoplsmisis 64. absoulte C/I in IUCD 65. 2 other q's about consent 66. Picture options:10 mcqs 67. 1 ECG:Mitral Stenosis 68. 2 ECG:LBBB 69. 3 Turner Sy -female genitalia 70. 4 Happy girl:school performance (no mental ret in Turner) 71. 5 Rectocel 72. 6 scleroderma 73. 7 Tempreture chart:ovulatory chart 74. 8 Kyst in the midle of the neck 75. 9 ptyriaqsis rosea 76. 1o Lesion on the lip:squamosus cell-carcinoma 77. Kid with migriane:Diagnostic, Treatement? 78. Why not give multivit to everyone.Which NOt true? Too Expensive 79. SSRi+ St John Wort? Serotonine syndrom 80. RTherapy + Hematuria? Cyctit Hem 81. When to start ABtherapy in C-section? After clamping 82. PCP intoxication 83. Ankle fracture + extremity cyanos.what to do? 84. Patient dies in emergency by a doc fault,but this doc is not employed by hosto and is Preiviliged??? who is responsilble?(Hosto+doc) 85. The most common cause of legal medical problems?Negligence 86. 14 years old wants OCP?Give to her 87. Mother wants to remove a normal beauty navi from 10 yo child`s face but the child wants to keep it.what u do? Respect child decision

1. Rheumatoid arthritis patient develops tinnitus. Cause? a) Due to ASA 2. Most common occupational disease in Canada. a) silicosis b) dermatitis c) asthma d) asbestosis 3. Most common cancer in Canada/ a) lung b) breast c) prostate d) brain

4. 23 year old lady presented with cramping abdominal pain and heavy bleeding. Pregnancy test positive. Bleeding stopped today morning. Pregnancy symptoms disappeared one week ago. USG shows empty uterus and 3 cm mass in adnexa. Diagnosis? a) ectopic pregnancy b) complete abortion with corpus luteal cyst c) ves. Mole with corpus luteum d) twin gestation- one aborted and one ectopic 5. A lady 50 years with swelling medial to femoral pulse. Firm and smooth. Diagnosis? a) femoral hernia b) inguinal hernia c) saphena varix 6. galactorrhoea . cause? a) pituitary infarction b) hyperthyroidism c) danazol d) thoracic nerve stimulation e) dopamine agonist 8. Most common gynecological infection in Canada a) candidiasis b) bacterial vaginosis c) gonorrhea 9. Most common cause for premenstrual itching (cyclical itching) a) candidiasis b) bacterial vaginosis c) non specific vaginitis 10. Benzodiazepam. Cross-tolerance with which drug a) chlorpromazine b) alcohol c) tricyclics 11. A child has ingested 30 tablets of imipramine. What should be done mean while contacting the toxicology center. a) forced diuresis b) give ipecac syrup c) drug to prevent convulsions 12. Which combination should not be used? a) tranyl cypramine-sertraline 13. Toxic shock syndrome. Not true a) exo toxin b) tampons c) skin necrosis d) fever e) rash 14. Commonest cause of broncheolitis RSV 15. A child with proptosis, swelling and redness around eye, movement of eye painful. Diagnosis a) orbital cellulites b) preorbital cellulitis 16. Soldier returning from peacekeeping force. He has nightmares anxiety. Etc. what is the initial step in management a) psychoanalysis b) benzodiazepine for 1year c) a trial of behavior therapy for s assort period 17. hirshsprungs most imp. Inv a) Barium enema b) Colonic transit time with radio opaque markers c) X ray 18. hirshsprung .diff from constipation a) from birth b) fecal soiling 19. Man 56 years. Father died of prostate ca. o/e a small nodule right lobe. PSA 2.2. What advice a) follow up in 6 months DRE and PSA

b) foloow up in 3 months PSA c) uss guided Biopsy now 20. severe premenstrual syndrome. Definitive management a) b/l oopherectomy b) hysterectomy c) prog d) oes e) NSAID 21. For which of the following is behavior modification is best? a) phobic anxiety b) generalized anxiet 22. A man 10 days after pancreatitis complains of abdominal pain, firm swelling a) pseudocyst, b) a/c hepatitis c) cholangitis d) 23. Peptic ulcer patient complaints of pain radiating to back. Diagnosis a) perforation b) penetration 24. A baby with birth wt.3.5kg now 4 weeks weighs 3.6 kg. Mother worried not drinking enough milk. What will you advise? 25. Ophthalmoplegia with pupillary sparing. Cause? a) diabetes 26. A lady gave birth to a cleft palate baby. You are going to send her home after counseling. For what reason she is going to come to your office more often? a) feeding problem 27. Which is not a cause for occupational asthma 28. A man with dark skin, liver 20 cm. Cirrhosis. Diagnosis a) hemochormatosis 29. What inv. In the above patient a) transferrin saturation 30. A man presented with pain in flanks radiating to groin. He had similar episodes 2 times previously. And was diagnosed to have hyper calciuria. What should be done? a) thiazides b) allopurinol c) reduce dietary calcium 31.post appendicectmy pt develop s-subphrenic abscess 32. Neonatal sepsis.a feature a) hypothermia 33. A man with a swelling in the right scrotum. O/e- tender mass above testis. skin of scrotum red and inflamed. Prostate enlarged He has pyuria. What is the diagnosis? a) epididymitis b) epididymitis and prostatitis c) gonococcal urethritis 34. How to look for clue cells. a) vaginal secretion with KOH and cover 35. Which is true about congenital anomalies? a) congenital anomalies rarely affect growth b) if symmetrical IUGR is detected look for congenital anomalies c) trisomy 13&18 will not cause growth retardation 36.esophageal atresia with tracheo esophageal fistula at the lower end. What is true? a) gastric aspiration common b) gastric air shadow not seen c) 37. A man sustained a stab wound on the thigh anteriorly. He is bleeding profusely. What is the first thing to do? a) apply tourniquet above the wound b) press femoral artery c) direct pressure d) on wound 38. A child with 15% burns On face and chest. What is done?

39. Laryngeal nodule- not a risk factormedication 39. A man 64 years c/o hoarseness. What is the first thing to be done? a) chest X ray b) laryngoscopy 40. Uncomplicated intestinal obstruction. All are seen except a) increased bowel sounds b) distension c) guarding d) tympanitic 41. Most common cause of syncope in a young man a) vasovagal b) hypoglycemia 42. Right upper quadrant pain, fever, jaundice. Diagnosis a) a/c cholecystitis b) cholangitis c) hepatitis 43. A/c cholecystis. Cause. a) obstruction of cystic artery b) onstruction of CBD c) obstruction of GB by stone 44. Treatment of nephritic syndrome. All except a) diuretics b) salt restriction c) fluid restriction d) prednisolone e) protein restriction 45. Malaria treated. Recurrence after 1 year a) dormant liver schizhont 46. Features of meningitis. What inv. a) lumbar puncture 47. Elderly wife abusing husband who is suffering from arthritis what is not done a) send hem to daycare b) support to wife c) 48. Status asthmaticus a) hydration, inhaled beta agonist, IV steroid 49. Abruption placenta. Cause of DIC a) fribrinolysis. 50. followings r characteristics of psychoanalysis except:

a) contertransference b) resistance c) rehearsal 51. newborn with congestive heart failure, is acyanotic, most likely cause? a) TOF b) ASD c) aoteric stenosis 52. scenario of vaginal candidiasis and groin skin candidiasis, the patient probably has: Answer: DM
community health 1-plants worker what cause most upsent ? 2-mortality rate decrease why ? better life style 3- who assess the capacity of pt 4- to refuse cont of resus who will sign psychiatry 1- lady afraid to give speech diffn ? social phobia 2-BN whats not tru metabolic acidosis

3-diffention of non social PD 4- difference between schizo and bi bolar disorder 5- < 24 mmse in elderly ? paeds: 1-all true regarding breast milk except 2-q's about achondroplasia risk in children mother not a carrier father not her brother is a carrier 3-UTI in child what Ix to be done MUCG + US 4-signs of mutism 5-autism = cont to stare at u ( good social ) 6- feature of leukaemia in a child = thrombocytopenia + neuotropaenia medicine 1- low mg all true except high ca 2-labetalol not used in 3-dka what not to do 4-most common hypothyroid disorder 5- best rx of dyspepsia 6- a case of hepatic encephalopathy 7- case of drug induced cirhosis 8- case of dermatomyocities 9- case old man obese snoring paq2=60 mg hg what advice 10- elderly afib adivese = start on coumadin therapy 11- case of mycoplasma pneumonia 12- which of the following organism doesnt caue lung abscess 13-rx of pleural effusion 14- man playing sea vollyball sudenly develop severe dyspnoa + mediastinal shift what to do = insertion of a chest needle ( spon.. pneumothorax) obs& gynae 1-case of PCOS 2- primgravida with active phase whats the feature 3- indication of assisted delivery except = station at -1 4- IUGR not associaed with =short lady 5- not true regarding karotype = umbilcal fetal blood smaple 6-variable deceleration= cord compression 7-most common cause of non reactive nst = fetal sleep 8-indication of parentral dx 9- f/u with a pt with GTN = serial bhcg surgery 1-man walks 2 block and rest = angioplasty 2- q's regarding inital Ix intermittent cludication = AB index 3- ulcer beside 2nd molar teeth elderly man with il-defined denture = trumatic ulcer 4- elderly with chronic constipation rx = enema 5- 2 q's about femoral hernia

MCCEE January 2004 1. Wrist drop, which nerve is affected? a) median nerve b) radial nerve c) ulnar nerve answer: b 2. Characteristic of borderline personality disorder is: Answer: splitting 3. Femur fracture with angulation, lower leg is pale, pulseless and cold, whats the first management? a) straighten the leg and traction b) immediate operation 4. In designing a community prevention program, which one should be undertaken first? a) obtain guarantee of funding b) estimate the cost-benefit of intervention c) quantify objectives d) carry out a community needs assessment e) formulate possible strategies of screening answer: d 5. All can cause depression except: a) cimetidine b) corticosteroids c) cephalosporines answer: c 6. cricothyroidectomy is indicated for a) foreign body in the trachea b) severe facial injuries c) C5-C6 spine injuries 7. child with yellow skin, which one is indication of carotenemia? a) no discoloration of the sclera 8. patient said his boss let him come to see u because he always cannot keep good relationship with his colleagues, he thinks his boss is harassing him, whats the probable diagnosis? a) borderline personality discorder b) dependent personality discorder c) schizotypal personality discorder

9. young lady with sudden onset right lower quadrant pain, pregnancy test is negative. Pelvic examination reveals 6cm severe tender right adnexal mass, diagnosis? a) adnexal torsion b) ruptured ovarian cyst c) ectopic pregnancy 10. old patient referred by his dentist with white lesion in his mouth, management? Answer: biopsy 11. old patient with whitish thickening lip, management? a) biopsy 12. child bitten by some kind of insect came with symptoms of anaphylactic shock, management? Answer: SC epinephrine 13. picture shown whitish lesions on the tongue (oral thrush) of an old woman after taking antibiotics for 10 days. Management? a) mycostatin gurgle b) coticosteroids c) amphotericin answer: a 14. all drugs r teratogenic except: answer: heparin 15. typical senario of trigeminus neuralgia, what do u give? Answer: carbamazepine 16. all of the following is true about incidence except: a) estimate the risk of acquiring the disease in community b) useful for etiology analysis for both acute and chronic diseases answer: b 17. most common irritant for child asthma at home: a) rugs b) pets c) plants d) smoking 18. best diagnosis of chronic pancreatitis: a) CT b) ERCP c) serum amylase answer: b

19. common symptom of cardial tamponade and pneumothorax: a) hypotension b) bradycardia c) decreased venous pressure answer: a 20. which of the folloing most suggests a pneumothorax? a) deviation of trachea b) paradoxical respiration 21. 30 y.o. female patient, her father has huntinton chorea, how big is the chance that she also has the disease? Answer: 50% 22. patient after big operation develops suddenly dyspnea, chest pain (probably PE), the most likely finding on chest x-ray is: answer: normal 23. patient with obstructive pulmonary disease, which of the following ABG finding suggests oxygen therapy may be dangerous: a) pH=7.34, pCO2=60, bicarbonate=30 b) pH=7.4, pCO2=40, bicarbonate=24 c) pH=7.19, pCO2=25, bicarbonate=forgotten answer: a (primary respiratory acidosis and compensation with metabolic alkalosis) 24. imperforated hymen is associated with all of the following except: a) abdominal mass b) limb paralysis c) obstruction of venous return d) hydronephrosis 25. police took a man who sexually assaulted a boy. When u asked him how many legs does the horse have, he answered 5. Dignosis? Answer: factitious disorder 26. followings r characteristics of psychoanalysis except: a) contertransference b) resistance c) rehearsal 27. 18 y.o. freshman came to u asking about the best way to prevent HIV transmission, your best answer is: a) combination use of condom and spermacide b) sexual abstinence c) judicious use of condom d) HIV test of partners

28. patient taking many drugs including ACEI, TCA and complaints of impotence, which drug is most likely to cause this symptom? a) ACEI b) TCA 29. female patients presents with palpitation, EKG shows occasional premature ventricular beats, management? Answer: reassurance 30. young lady complaints of hopelessness, decreased in concentration especially at the end of the day. management? a) time off work b) antidepressant c) psychosocial support and return visit d) psychosocial support and lorazepam 31. young female patient with genital herpes, what do u suggest? a) she can not conceive in the future b) she can conceive but she should tell her obstetrics about her herpes history answer: b 32. newborn with hart rate 40/min, irregular respiration, some flexion of the extremities, grimace to nasal catheter. He is pale and blue. Apgar score is? a) 2 b) 3 c) 4 d) 5 e) 6 33. newborn with congestive heart failure, is acyanotic, most likely cause? a) TOF b) ASD c) aoteric stenosis 34. patient on warfarin for phlebitis presents with spontaneous bruises and petechiae. INR is also increased. Management? a) reduce warfarin b) stop warfarin c) stop warfarin + vitamin K d) stop warfarin + fresh frozen plasma 35. scenario of endometriosis, definitive diagnostic method? Answer: laparoscopy

36. female patient presents with right lower quadrant pain. Now she also complains of epigastric and left abdominal pain. On examination, she has tenderness on epigastrium and left abdomen. Fever 38C, labor: your management within 30 min includes all of the following except: a) repeat abdominal examination b) check BP and pulse on standing c) pelvic examination d) requisition of ultrasound which is scheduled in next week 37. one disease family tree, ask what inheritance type a) autosomal recessive b) autosomal dominant c) X linked recessive d) X linked dominant e) multifactorial 38. patient on lithium presents with fatigue, weight gain and constipation. Investigation? a) serum electrolytes b) serum lithium level c) TSH answer: c 39. black woman, nullipara, jogs 30 minutes three times a week. She smokes for 20 years. Height 150cm, weight 50kg. which of the factors increase the osteoporosis? a) her race b) her BMI c) exercise d) smoking e) nullipara answer: d 40. an evidence of decreasing osteoporosis in post-menopausal woman who are taking calcium supplement can be strongest obtained from a) a randomized control trial of giving calcium tablets and follow up for fractures for 10 years b) comparison of fracture incidence in postmenopausal women who are living in high and low diary products available answer: a 41. the most common cause of absence from work is answer: back strain 42. most important bias factor in cohort study is a) follow up b) recall

answer: a 43. patient presents with hematuria and albuminuria. Which test can confirm the diagnosis of acute glomerolonephritis? a) abdomen US b) ASO titre 44. all of the following are associated with posterior hip dislocation except: a) avascular necrosis of femoral hear b) sciatic nerve injury c) femoral nerve injury 45. your colleague was preparing to do a surgery. You found out his gait is abnormal and you got smell from him. You let him not to do the surgery but he said it was not your business. What should you do? a) inform the chief of staff on the next morning b) inform the CEO immediately c) let another colleague help to talk to him 46. patient on clozapine, important side effect of this drug: answer: agranulocytosis 47. pap smear is a) primary prevention b) sencondary prevention c) tertiary prevention answer: b 48. scenario of acute pyelonephritis, the most common cause is: a) staphylococcus aureas b) E. coli c) Chlamydia trachomatis answer: b 49. woman with breast cancer was treated with radiotherapy. She was responding well but developed depression. Then, she got pneumonia and died. What is the cause u should write on her death certificate? a) pneumonia b) depression c) brest cancer answer: a 50. your neighbor called u because their child was choking during eating. U should do: answer: put the child facing down and give blows on the back at the interscapular area

51. best treatment of bullous impetigo: a) penicillin b) amoxicillin c) cloxacillin 52. patient with lower back pain, x-ray: anterior osteophytes at L4-L5 and subchondral sclerosis, diagnosis: a) degenerative spondylitis 53. middle aged female patient with gastroesophageal reflux complaints of dysphagia for liquid and solid food. Cause? a) barrett esophagus b) squamous cell carcinoma of esophagus c) adnocarcinoma of esophagus d) peptic stricture 54. picture shown 6-month-old infant with umbilical hernia. Your management? Answer: reassurance and recheck at 1 year of age 55. in postoperative period, a patient demanded for morphine for relief of his pain every 4 hours. The attending nurse thought that this patient was lying and told you that the patient should be treated with placebo. What will u do? a) give placebo b) not give placebo c) tell the patients relatives for consent to give placebo 56. U are family physician of a lab technician. He always drinks. U will report all the conditions except: a) you know that a lab technician disturbs his lab works b) he drives car during drunk c) he occasionally drives his private airplane during drunk d) has sexual relationship with his 14-year-of daughter 57. patient presents with recurrent gouty arthritis attack, urinary uric acid is increased, you will prescribe: a) colchicine b) allupurinol + colchicines c) benzbromaron 58. patient post-MI, develops bradycardia with pulse 32/min, the best management a) atropine b) pacemaker answer: b 59. scenario of neuroleptic malignant syndrome, treatment?

a) benzodiazepine b) bromocriptine c) benztropine d) buspirone answer: b 60. which neurotransmitter in schizophrenia? Answer: dopamine 61. definition of ergonomics? Answer: concerned with human factors in the design and operations of machines and the physical environment 62. patient postoperative presented with heavy wound bleeding, she also gave a history of massive bleeding when she had a tooth extraction. Blooding time is increased. You will give: a) DDAVP b) factor XIII c) fresh frozen plasma answer: a 63. KOH is used to diagnose: a) candidiasis b) bacterial vaginosis c) thichomoniasis answer: a 64. young male patient with painless scrotal mass, transilumination ive, management? Answer: refer to surgery 65. hepatitis B can be transmitted through the following ways except: answer: fecal-oral 66. the best screening test of primary hyperaldosteronism in hypertension: answer: serum rennin activity level 67. male patient had problem at work. Now he complains of back pain. Physical exam shows no pathological finding but he obsessed with it. Diagnosis? Answer: malingering 68. patient with deceitfulness, impulsivity and lack of remorse. Diagnosis? a) antisocial PD b) borderline PD 69. boy fell from a tree, presented with abdominal pain and hematuria. U will do:

a) renal scan b) CT note: there were no option like IVP so I chose CT 70. central tendency: mean systolic pressure: 115mmHg with standard deviation of 15mmHg, ask 95% of the population will have SP of: Answer: 85-145mmHg 71. incidence of a disease in the exposure group is 137, in the non-exposure group is 8, ask 137/8 is called: Answer: relative risk 72. decreased mortality rate of stroke is due to: a) better control of blood pressure b) regular use of aspirin c) improvement of surgery process 73. patient treated with amoxicillin for 10 days developed diarrhea, how to confirm your diagnosis? a) stool culture for clostridium dificile b) coloscopy c) test of clostridium dificile toxin 74. newborn needs: a) 75 kcal/kg/day b) 115 kcal/kg/day c) 200 kcal/kg/day answer: b 75. female with adenomyosis is likely to present with Answer: bulky uterus 76. scenario of angle closure glaucoma, your first management: a) miotic eye drop b) mydriatic eye drop Answer: a 77. 75 years old man presented with bilateral loss of vision, he gave history of herpes zoster. What is the most likely cause of his visual loss a) bilateral corneal opacity due to herpes zoster b) macular degeneration c) glaucoma Answer: b 78. post trauma, which of the following signs enforces you to admit the patient? a) corneal abrasion

b) conjunctival tear c) soft eye answer: c 79. shock baby syndrome answer: you may find intraocular hemorrhage 80. picture shown clubbing fingers, which is NOT the cause? a) lung cancer b) chronic bronchitis c) infective endocarditis 81. child with 2 years history of tourettes syndrome, now developed obsessive compulsive disorder, what would your give? a) haloperidol b) fluvoxamine 82. which cancer can caused by vinyl chloride? Answer: liver CA 83. which one is not a symptom of lead poisoning? Answer: liver cirrosis 84. which of the following trauma must be treated first? Answer: pneumathorax 85. picture of female newborn shown vaginal bleeding, management? Answer: reassurance 86. patient with anxiety, dysphoria, muscle pain, accasionally convulsion, these are the withdrawal symptoms of which of the followings? a) benzodiazepine b) cocain c) cannabis 87. the difference between male and female sexuality is: Answer: female has shorter refractory period 88. most common source of silicosis exposure is: Answer: sandblasting 89. 12 months infant only with breast feeding till now is most on risk of what deficiency? a) vitamin D b) iron c) folic acid

90. child with knee pain, swelling and redness of the knee joint, management? Answer: knee aspiration 91. Downs syndrome is associated with the followings except: Answer: rocker bottom foot 92. patient had barbecue, developed muscle pain, diarrhea, eosinophelia. Best investigation to confirm the diagnosis? Answer: muscle biopsy 93. scenario of vaginal candidiasis and groin skin candidiasis, the patient probably has: Answer: DM 94. patient presented with inability to dorsiflex the foot and loss of sensation over the dorsum of the foot, diagnosis? a) peripheral neuropathy b) disc compression Answer: b 95. conversion disorder, all are true except: a) la belle indifference b) unconsciousness of the symptoms c) defence mechanism of projection and intellectualization 96. female vaginal lubrication during sexual excitement period is due to secretion of: a) cervical glands b) vaginal glands c) vaginal transudat

1-A 60 y old woman presents to your office for her annual check up. Because of a long smoking history you order a chest X-ray. The radiologist phones you the next day to report that there is a new 1cm nodule in the right upper lobe. What action would you take in the management of this patient? a. Do nothing at this time as the nodule is asymptomatic b. Do a TB skin test c. Arrange for a biopsy d. Repeat the chest x-ray in 6 months e. Inform her that she has cancer 2-A patient presents with a winged scapula (posterior flaring of the tip of the scapula when attempting to push against a wall). Which of the following nerves has been injured? a. Long Thoracic Nerve b. Thoracodorsal nerve c. Suprascapular nerve d. Upper subscapular nerve e. Lower subscapular Nerve 3. An 18 Y old male driver,involved in a motor vehicle account, arrives in the ER complaining of left sided chest pain and dyspnea.On examination HR is 140, BP is 70/50 and RR is 36.He has decreased breath sounds in the left hemothorax, his trachea is shifted to the right and there is increased resonance to percussion over the left hemothorax. Immediate management /of this patient would require: a. Oxygen 5 l/min NP b. 2 large bore IV`s c. Morphine 10mg/min d. 14 gauge needle left 2nd intercostal space followed by a chest tube e. Left chest tube 4.When contracted by an infant, which form of hepatitis has the highest rate of chronic carriage? a. hepatitis A b.hepatitis B c.hepatitis C d.hepatitis D e.hepatitis E 5. A 5 y old boy develops erosions of the lips ,conjunctivitis,fever and a blistering rash.He has just completed 10 days of therapy with cotrimazole (septra or Bactrim) for otitis media. The best diagnosis for this disease is: a. Stevens-Jonhson Syndrome b. Varicella c. Rubeola d. Lichen Planus e. Candidiasis

6.Which of the following characteristics would be helpful in differentiating attention deficit hyperactivity disorder from conduct disorder in children? a. Resistance to discipline b. Temper tantrums c. Distractibility d. Aggressiveness e. Truancy 7.A 3 y old black girl from Haiti experiences a cerebrovascular accident. CBC reveals : Hb 7.5 , MCV 82, WBC 8000 , platelet 450,000. Which of the following tests is most likely to establish the diagnosis? a. ANA titre b. Echocardiography c. EEG d. Hemoglobin eletrophoresis e. Lumbar puncture 8. A 45 y old male is brought to the ER by his family,having just coughed up approximately 2 cups of bright red blood. As you arrive to assess him, he coughs up more blood ,half filling a wash basin. Your next step would be: a. Call to arrange for a flexible bronchoscopy b. Transfuse 2 units packed red cells c. Start Oxygen d. Arrange an angiogram e. Take him immediately to Operating room for a rigid bronchoscopy 9.The natural history of a child with infantile spasms is most likely to include: a. Development delay b. Growth arrest c. Infantile autism d. Visual deficits e. Remission of the seizure-disorder 10.A 62 y old man has a 6 months history of intermittent pain, stiffness and swelling of the right knee. Examination revealed a small knee effusion with coarse crepitus, medial joint line tenderness and limited flexion to 100 degrees.Synovial fluid aspirated from the Right knee is clear and shows a total leukocyte count of 1.4x10 /L with 20% of neutrophils.The fluid is sterile on Culture and there are no crystals.Which of the following is the most likely diagnosis? a. Osteonecrosis of medial femoral condyle b. Monoarticular Rheumatoid arthritis c. Exacerbation of knee osteoarthritis d. Chronic Pyrophosphate arthropathy e. Monoarticular-onset seronegative spondyloarthropathy

11.A 3-day old infant is unable to raise either upper eyelid. Which of the following is the most likely diagnosis? a. Clostridium Botulinum Infection b. Congenital Ptosis c. Intraorbital Tumour d. Myasthenia Gravis e. Congenital hyperthyroidism THE FOLLOWING STEM PERTAINS TO QUESTIONS 12 AND 13 A 75 y old man presents with acute mono-arthritis of right wrist of 2-day duration.There is no history of antecedent trauma or previous arthritis. Radiographs show calcification of the triangular ligament (disc) of the right wrist. Aspiration of the joint yelds one ml of cloudy yellowish synovial fluid..Examination of the fluid under compensated polarized light microscopy shows weakly positive-birefringent rhomboid-shaped and pleomorphic crystals. 12. The crystals are most likely: a. Monosodium Urate b. Cholesterol c. Calcium Hidroxyapatite d. Dicalcium phosphate dihydrate (brushite) e. Calcium Pyrophosphate dihydrate 13. Which of the following is the most likely diagnosis? a. Septic arthritis of right wrist b. Acute Pyrophosphate arthritis (acute pseudogout) c. Acute urate (gouty) arthritis d. Acute calcific periarthritis e. Acute exacerbation of radiocarpal (wrist) osteoarthritis (end of the series) 14. Breast feeding is a contraindication to which of the following vaccinations? a. Diphtheria b. Influenza c. MMR d. Hepatitis B e. None of the above 15. A 40 y old woman has a 3 year history of an erythematous rash over scalp, ears,elbows, legs and feet.She presents with a 6-month history of pain and stiffness of the lower lumbar spine, right wrist, left knee and both ankles.She has multiple scaly erythematous skin lesions over elbows,scalp and soles ; and pitting of fingernails.Lumbar spine movements are restricted and she has an asymmetric chronic inflammatory oligoarthritis of the right wrist,left knee , both ankles and distal interphalangeal joint of left middle finger, and a sausage-like swelling of the right 4th toe. Results of laboratory studies: Hb 135 g/L, total leukocyte count 6.5x10 (9)/L with a normal differential, ESR

30mm/h, negative rheumatoid factor and posiutive HLA-B27. Which of the following is the most likely diagnosis? a. Seronegative rheumatoid arthritis b. Psoriatic arthritis with spondylitis c. Ankylosing spondylitis d. Reactive arthritis (Reiter`s Syndrome) e. Sarcoid arthritis 16.In a 24h urine collection,the most common lab abnormality associated with renal calculi in patients under the age of 40 years old is: a. Hypercalciuria b. Hyperoxaluria c. Low Citrate level d. Decreased urine volume e. Proteinuria 17. A 15 y old with Crohn`s disease involvimng the terminal ileum and colon develops hematochezia, abdominal cramps and tenesmus. Which of the following s the most effective treatment of these symptoms? a. Azathioprine b. Corticosteroids c. Cyclosporine d. Metronidazole e. Sulfosalazine 18.A 3 y old has recurrent urinary tract infections. Voiding cystourethrogram reveals grade 2 vesicoureteral reflux. The best course of action is: a. Obtain urine cultures weekly b. Obtain radionuclide cystograms every 6 months c. Prescribe prophylatic antibiotic therapy d. Refer for surgical correction of the reflux e. Test urine daily at home with nitrate dip sticks 19. A 60 y old male who is a factory worker in a rubber tire company, presents to ER with an inability to void. His history of present illness recounts that he noticed painless gross hematuria approximately 2 days ago. One day ago he found it difficult to empty his bladder with the passage of clots. On physical examination, BP is 120/70, HR is 120. He is visibly in marked distress. On abdominal examination, his bladder is palpable above the umbilicus.Hemoglobin is 120, white count is 14 and platelets 370. In the initial management of this patient you would: a. Insert a suprapubic cystotomy b. Perform a laparotomy and cystotomy to stop the bleeding of the bladder c. Insert a 18 french 2-way catheter and try to empty the bladder d. Insert a large bore 3-way catheter to evacuate all clots within the bladder and start a continuous bladder irrigation

e. Bring the patient to the operating room for an urgent cystoscopy and avacuation of clots under general anaesthesia. 20- REQUIRES FIGURE (NOT POSSIBLE TO SEND) 21- If a traveler`s diarrhea lasts for more than 14 days it is considered to be persistent.What is the most common class of micro-organisms that causes this type of persistent diarrhea? a. Gram negative bacteria b. Gram positive bacteria c. Protozoa d. Viruses e. Helminths 22- The most common cause of post menopausal bleeding is: a. Endometrial carcinoma b. Ovarian carcinoma c. Foreign body d. Atrophic vaginitis e. Cervical cancer 23-which of the following is a correct statement about cosleeping (children sleeping with their parents during the night) ? a. Children who sleep with their parents are more likely to sleep through the night b. Cosleeping encourages independence and autonomy in the child c. Cosleeping is less common in black families than in Caucasian families d. Most pediatricians support cosleeping in the preschool child e. Most toddlers sleep with a parent at least once a month 24- A 26 y old primigravida presents at 40 weeks in active labour with contractions every 2 minutes.She is diagnosed as having a transverse lie with the back up. Which of the following would be most appropriate next step? a. Start Vasodilan (Isoxuprine) b. Perform an external version c. Prepare for an immediate C-section d. Rupture membranes and then perform and internal version e. None of the above 25. A 34 y old homosexual male presents with chest pain.He had oral trush 3 years previously, and he had been reasonably well since.He has been maintened on AZT, sulfamethoxazole/trimethroprim, ketoconazole and Imodium. However, he had felt that the sulpha gave him a little rash, so he stopped it 2 months earlier.He had lost about 8 pounds in weight over the past year. For almost the same period, he had noted some night sweats, cough and malaise.The current illness had slowly crept up on him, with a gradual, perharps 4 weeks onset of central chest pain. The pain does not increase with respiration, although there is mild accentuation with cough.When examined , the patient looks ill.Chest exam is negative, except for a few wheezes.lab values include:Hb95,WBC

3.2,Gluc 5.8. Arterial blood gasses (room air): PH 7.46, PCO2: 35, PO2: 66. Which of the following is the most likely diagnosis? a. P.carinii pneumonia b. Cryptosporiium diarrhea c. Mycobacterium avium complex d. Pulmonary Kaposi sarcoma e. Staphylococcus pneumonia 26.A 2 y old has a history of recurrent pneumonia, short stature and failure to thrive. Studies reveal neutropenia,thrombocytopenia,normal sweat chloride and metaphyseal dysostosis of the head of left femur.What is the most likely diagnosis? a. Sideroblastic anemia b. Trypsinogen deficiency c. Wiskott-aldrich syndrome d. Schwachman-Diamond syndrome e. Cystic fibrosis 27.Which of the following treatments is most likely to be helpful for attention deficit hyperactivity in a 10-year old child? a. Behaviour modification b. Biofeedback c. Hypnosis d. Modified diet e. Vision training 28.The gastric neoplasm most known to bleed massively is: a. Adenocarcinoma b. Leiomyoma c. Carcinoid d. Reticulum Cell sarcoma e. Lymphoma 29.A child born at 32 weeks gestation is brought to your office at 2 months of age for a routine check-up. The child has an upper respiratory tract infection but is afebrile.What decision would you take regarding the child`s first vaccination? a. Wait until the child is 2 months older to administer the first vaccination b. Wait until the child is clinically well and reschedule the vaccination c. Administer the first vaccination d. Consult a Pediatrician e. None of the above 30.A 1 year old child has a red scaly rash involving the scalp, arms and legs.There is no family history of allergy. The rash is characterized by discrete plaques and has no cleared on over-the-counter hydrocortisone cream.Which of the following is the most likely diagnosis? a. Psoriasis b. Seborrheic dermatitis c. Seborrheic keratosis d. Keratosis Pilaris e. Lupus erythematosus

ANSWER KEY 1-30: 1- C 2- A 3- D 4- B 5- A 6- C 7- D 8- E 9- A 10- C 11- B 12- E 13- B 14- E 15- B 16- D 17- B 18- C 19- D 20- NOT SHOWN 21- C 22- D 23- E 24- C 25- A 26- D 27- A 28- B 29- C 30- A 31-Which of the following is a correct statement about the anatomy of the female breast? a. The lymphatics drain entirely into the ippsilateral axilla b. Its embryologic origin is from ecto, endo and mesoderm c. The sensory supply to the nipple is from the T2 nerve root d. It commonly includes a tail of breast tissue which extends along the pectoralis muscle towards the axilla e. The embryologic milk line is lateral to the anterior axillary line 32-A 25 year old primigravida at 34 wekes of gestation is thought to be small for dates by her physician and is sent for a Sonographic evaluation. The ultrasound shows the Biparietal diameter to be appropriate for 34 weeks of gestation. The abdominal circunference is appropriate for 30 weeks gestation.The head:abdominal circumference

ratio is 1 .The estimate fetal weight is 10th percentile for 34 weeks gestation.The amniotic fluid is decreased.Which of the following is the most likely diagnosis? a. Symmetrical IUGR b. Asymmetrical IUGR c. Congenital anomaly d. Congenital infection e. Unknown gestational age 33.A common side effect of the Serotonin re-uptake inhibitors (SSRI) is: a. Increased appetite and weight gain b. Dry mouth and constipation c. Cardiac conduction defects d. Sexual dysfunction e. Postural hypotension 34-Not shown 35-A 5 year old child develops the sudden onset of a bright red rash on the checks and a fine net-like rash on the tighs. The child is otherwise well with no fever or other symptoms.Which of the following is the most likely causative agent? a. HIV b. Enterovirus c. Paxvirus d. Parvovirus e. Myxovirus 36-A motor vehicle crash victim has a wide mediastinum with an indistinct aortic knob on an upright plain chest radiograph. The patient is hemodynamically stable.Which of the following is the diagnostic test of choice? a. CT scan of the chest b. Repeat chest radiograph in 2 hours c. Aortogram d. Transthoracic echocardiogram e. Broncoscopy 37-Which of the following statements is correct about alcohol withdrawal seizures? a. They occur 4-7 days after drinking stops b. They can be prevented by prophylatic treatment with thiamine c. They are usually the first symptom of delirium tremens d. The patient will require long term treatment with anticonvulsivants e. They can be prevented by stimulation of gamma amino butyric acid receptors in the brain 38-A 43 y old man admitted for emergency gastrectomy develops confusion on the third post-operative day.He complains of lack of sleep due to cats in his hospital room all night and complains also of cockroaches on the ceiling.He is noted to be flushed and tremulous by the nurses during the day.The most likely problem is: a. Post-operative electrolyte imbalance b. Paranoid schizophrenia c. Depressive psychosis d. Delirium tremens

e. Anoxic brain syndrome 39-Life expectancy for Canadian women is known to be : a. Lower than that for Canadian males b. More adversely affected by premature mortality compared to Canadian males c. At the biological limit of about 105 years d. Positively related to socio-economic status as measured by annual income e. Influenced substancially by maternal mortality rates 40-Not shown 41- Which of the following characteristics of a diagnostic test is influenced by the prevalence of the disease being tested for? a. Specificity b. Sensitivity c. Accuracy d. Positive predictive value e. Reliability 42-Congenital infection with Toxoplasma Gondii or cytomegalovirus is most likely to be associated with: a. Chorioretinitis b. Chronic active hepatitis c. Congenital heart disease d. Congenital pneumonia e. Spastic diplegia 43-Which of the following would be the recommended formula for a newborn whose parents both have asthma? a. Soybean formula b. Whey hydrolysate formula c. Cows milk formula with iron d. Cows milk formula without iron e. Goats milk 44-Sudden headache with vomiting and an inability to sit up is most likely : a. Meniere`s disease b. Migraine c. Positional vertigo d. Vestibular neuronitis e. Cerebellar hemorrhage 45-In the differential diagnosis of ectopic pregnancy,ultrasound is used primarily to: a. Identify bleeding in cul-the-sac b. Identify intrauterine gestational sac c. Identify adnexal mass d. Measure size of uterus e. Estimate gestational age 46-Sudden loss of vision in the left eye that resolves after 5 minutes could be due to: a. Left Carotid artery stenosis b. Glaucoma

c. Optic neuritis d. A right occipital lobe lesion e. A left occipital lobe lesion 47-The single most important determinant of gastro-esophageal reflux is: a- The PH of gastric contents b- Hiatal hernia c- Esophageal peristalsis d- Loss of the intra-abdominal portion of the esophagus e- Loss of lower esophageal sphincter tone 48-Not shown 49-Not shown 50-Which of the following is the best screening test for the presence of hemochromatosis? a. Percutaneous liver biopsy b. Serum ferritin level c. Family history of early-onset cryptogenic cirrhosis in first degree relatives d. Percent iron saturation in serum e. Alpha-fetoprotein level in serum 51-A 32 y old GO PO has been having regular unprotected intercourse for the past 3 years. She has severe pain during menses which radiates into her anal region.On examination a small,retroverted uterus is noted.There are nodules presented in the uterosacral ligaments and rectal vaginal septum.Which of the following is the most likely diagnosis? a. Uterine myomas b. Adenomyosis c. Endometriosis d. Chronic PID e. Ovarian Carcinoma 52-Which of the following is a sign of severe preeclampsia? a. BP elevation 140/90 b. Edema c. Proteinuria 200 mg/24hours d. Trombocytopenia e. Dizziness 53-The most common site for an ectopic pregnancy is: a. Isthmus b. Ampulla c. Fimbria d. Ovary e. Interstitium 54-An 8 year old had a tonsillectomy and adenoidectomy 10 days ago. She has complained of pain in the right ear for the past week.Examination of the ear is normal.Which of the following is the most likely diagnosis? a. Infected tooth b. Nasopharyngitis as a complication of the adenoidectomy

c. Referred pain from the pharynx d. Subclinicfal otitis media due to edema of the Eustachian tube orifice e. Temperomandibular joint pain referred to the ear 55-A 3 y old has a ventriculo-peritoneal shunt inserted at 2 days of life for hydrocephalus. He develops vomiting and irritability.What is the best indication that the shunt is blocked? a. 4 by 4 cm anterior fontanelle b. Cranial bruit c. Dullness to percussion of the skull d. Pallor of optic discs e. The sun settingsign 56-Which of the following is an absolute contraindication of the MMR vaccine? a. Positive tuberculin skin test b. Infection with HIV c. Pregnancy d. Breast feeding e. Known anaphylactic reaction to neomycin 57-Which the following is the best plan for Influenza vaccination for a 9-month-old child with cystic fibrosis? a. Administer 1 dose of split virus vaccine now b. Adminster 1 dose of split virus vaccine now and again in 1 month c. Administer 1 dose of whole virus vaccine now d. Adminster 1 dose of whole virus vaccine now and again in 1 month e. Withhold virus vaccine until the child is 12 months old 58-At which stage of gestation should Rhogan be administered to an Rh negative female whose partner is Rh positive? a. 5 to 7 weeks b. 10 to 12 weeks c. 18 to 20 weeks d. 28 to 32 weeks e. Only at post delivery 59- In managing patients with type II diabetes , sulfonylureas and biguanides may be used.Which of the following are common properties of these drugs? a. Both raise insulin levels b. Both decrease insulin resistance c. Both may be given orally or intravenously d. Both lower ketone levels e. Both decrease HDL-cholesterol 60-The component of Canada`s health care system that receives the highest percentage of the health care budget is: a. Hospitals b. Physician fees c. Drug benefit plans d. Laboratory services e. Administration

61-A 22 y old sexually active woman presents with a 3 day history of fever, chills, multiple pustular skin lesions on arms,hands and legs,vaginal discharge and acute arthritis of right elbow,both knees and left ankle.Which of the following is the most likely diagnostic consideration? a. Reiter`s syndrome b. Gonococcal arthritis c. Bacterial endocarditis d. Vasculitis due to Henoch-Schoeinlein purpura e. Acute rheumatic fever 62-Which of the following presentations should be delivered vaginally? a. Persistent brow presentation b. Transverse lie (shoulder presentation) c. Footling breech presentation d. Vertex presentation with umbilical cord prolapse e. Face presentation (mentum anterior) 63-Which of the following pulmonary function tests most reliably discriminates pure Chronic Bronchitis from Emphysema? a. Total lung capacity b. Functional residual capacity c. Residual Volume d. Single breath diffusing capacity e. Flow at 50% vital capacity 64-At a meeting of the medical staff in your local hospital,a clinician who has been examining the patterns of mortality in your community says that rates for heart disease and lung cancer are higher in this county than in an adjacent county.Which of the following questions should you ask first? a. How did the clinician choose the comparision county? b. Have the rates been standardized for age? c. Are tabacco sales significantly different in the two counties? d. Are the facilities to treat these diseases comparable in the two counties? e. Are the numbers of deaths comparable in each county? 65-A woman complains of a copious whitish discharge which is non-irritanting, and has no odour.PH is 4.5. Microscopic examination shows many bacteria and epithelial cells.The best course of treatment would be: a. Clotrimazole (Canesten) b. An oral course of metronidazole (Flagyl) c. Reassurance that this is a normal finding d. Treatment for Inflammatory pelvic disease e. Local estrogen therapy 66-An 8 month old has a head circumference above the 97th percentile.Which of the following is the best diagnostic test to confirm hydrocephalus? a. brain scan b. EEG c. X-ray of the skull d. Transillumination e. Ultrasonography

67-A 50 y old male presents to your office with a 3 4 week history of fatigue and increasing dyspnea.Over the past week he has noticed that his face and arms are swollen.On examination you note that his face is puffy,his complexion is somewhat plethoric,his arms are swollen and there are proeminent veins over his anterior chest.There is a 2cm supraventricular lymph node but no other adenopathy in the neck, axillae or groins.Respiratory,CV and abdominal examination are unremarkable.You order a chest x-ray which reveals a widened mediastinum and a likely anterior mediastinal mass.At this point,appropriate management should involve: a. Diuretics to reduce facial swelling b. CT scan of the chest and an invasive procedure to biopsy the mass c. Immediate surgical resection d. Mestinon therapy e. Radiation therapy 68-A 32 y old G5 P4 presents with an 8 week history of amenorrhea and suggestive symptoms of pregnancy.Physical examination reveals an irregular enlarged uterus of 16 weeks size.Ultrasond examination confirms the presence of an 8 week viable pregnancy and a multiple fibroid uterus.The correct management is: a. Termination of pregnancy with elective myomectomy 2 months later b. Termination of pregnancy with concomitant myomectomy c. Prudent observation with elective caesarean section at term d. Prudent observation anticipating probable vaginal delivery e. Myomectomy and follow pregnancy in usual way 69-A 50 y old male who has osteoarthritis of the knee undergoes a high tibial osteotomy that is secured with staples. 8 hours later,the patient has severeleg pain that is not relieved with analgesics.On examination,the patient is unable to dorsiflex his toes.The limb is elevated , but symptoms persist 2 hours later.The most likely diagnosis is: a. Acute Trombophlebitis b. Hemorrage into the popliteal space c. Anterior Compartment syndrome d. Arterial embolus e. Loss of fixation and displacement of the osteotomy 70-A patient with anorexia nervosa: a. Believes deep down that she is underweight b. Usually avoids excessive exercise c. Will probably not resume normal menses when she recovers d. Measures successin terms of not eating and weight lost e. Often develops purging later in the course of the illness 71-Which of the following factors is most likely to explain the recent decline in ischemic heart disease and stroke mortality rates in North America? a. Major advances in surgical techniques to undo the effects of atherosclerosis b. Legislation controlling industrial pollution and restricting areas where cigarettes can be smoke c. Changing age composition of the population d. Major advances in drugs which can control hypertension e. Improved lifestyle habits in much of the population

72-A 34 y old man with long standing Crohn`s disease is convalescing after surgical resection of 50cm of terminal ileum,except that he complains of persisting diarrhea.The most likely cause is: a. A recurrence in Inflammatory bowel disease b. Malabsorption of bile salts c. ncoveredceliac sprue d. Pseudomembranous colitis e. Short gut syndrome 73-In 1991,which of the following was the leading cause of potential years of life lost(PYLL) in males aged 25-44? a. Motor vehicle accidents b. Cancer c. Suicide d. AIDS e. Heart disease 74-The most common sexually transmitted disease in females aged 15-24 is: a. Syphilis b. Gonorrhea c. HIV d. Chlamydia e. Chancroid 75-Which of the following is correct about child abuse? a. Children don`t lie about sexual or physical abuse b. The parents were often victims of abuse themselves c. It occurs mainly in the lower socio-economic classes d. The abuser is usually somebody the child doesn`t know e. You should not report your suspicions to the Children`s Aid Society unless there is ample evidence of abuse 76-Which of the following definitions best captures the meaning of incremental cost effectiveness analysis a. The cost of one treatment compared to the cost of another b. The cost of the treatment under study compared to the cost of an alternate, multiplied by the number of people who are eligible for the study treatment c. The difference in cost between the 2 alternatives treatment strategies ,divided by the gain in health outcomes achieved by adopting the new strategy d. The cost of the treatment under study minus the cost of the alternative treatment, divided by the average life expectancy of persons treated under the new regimen e. The increase in the cost of treatment compared to last year 77- A neonate with Coxsackie virus infection is most likely to have which of the following? a. Herpangina b. Hand-foot mouth disease c. Peritonitis d. Sepsis e. Pleurodynia

78-A 66 year old woman presents with a 3 months history of fatigue, anorexia, nausea,weight loss and vague abdominal pain.Her waistline increased during that time and now she requires a larger dress size.Pertinent physical findings include ascites and a left ovarian cyst measuring 8cm in diameter.Ultrasound shows this cyst to have septations, the uterus is small and the right ovary cannot be visualized. Ca125 level is 100. Which of the following is the best provisional diagnosis? a. Carcinoma of the fallopian tube b. Carcinoma of the ovary c. Cystadenoma of the ovary d. Carcinoma of the pancreas with pelvic metastasis e. Krukemberg tumour 79- A 3 y old is admitted with acute status asthmaticus.He receives theophylline IV ,ventolin inhalations and IV steroids. Four hours after starting treatment he suddenly develops marked agitation,flushing and a heart rate of 210/min. He is confused and incoherent.Which of the following is the most likely diagnosis? a. Beta-2 agonist toxicity b. Hysterical reaction c. Steroid psychosis d. Theophylline toxicity e. Acute hypercapnia 80-Which of the following is correct about depression in children? a. Family therapy should be avoided because it scapegoats a child who is already vulnerable b. Symptoms may manifest as antisocial behaviour c. Antidepressants generally are not effective in children d. The suicide rate in children aged 8-13 is higher than it is in older adolescents e. Depression in children has been shown to be a prodrome of the later development of schizophrenia 81-Olygohydramnios is associated with which of the following? a. Anencephaly b. Hydrocephaly c. Diabetes mellitus d. Renal agenesis e. Esophageal atresia 82-Which of the following organisms causes Lyme disease? a. Treponema Pallidum b. Yersinia Pestis c. Borrelia Burgdorfeii d. Salmonella typhi e. Rickettsial species 83-Erythromycin is the most appropriate treatment for which one of the following conditions? a. Cystitis b. Impetigo c. Otitis media

d. Shigellosis e. Sinusitis 84- A 45 y old man has intermittent dysphagia and occasional episodes of regurgitation of bland tasting food eaten several hours earlier.Which of the following would be the most appropriate procedure to yield a diagnosis? a. Barium x-ray of the esophagus b. 24 h PH monitoring c. Esophageal manometry d. Gastroscopy e. Acid perfusion test (Bernstein test) 85-The most common cause of post-partum hemorrhage is: a. Laceration b. Retained placental fragments c. Placenta accreta d. Uterine atony e. Distended bladder 86-A 13 y old male was running a track and field race when he developed sudden pain in the right knee.He had difficulties completing the race.A painful limp persisted for 3 weeks before he presented to your office.Physical examination revealed a tredelemburg gait and decreased internal rotation of the right hip.Which of the following is the most likely diagnosis? a. Osteonecrosis of the femoral head b. Slipped capital femoral epiphysis c. Iliopsoas hematoma d. Avulsion of the biceps femoris from the ischium e. Gluteus medius strain 87-Which of the following presents the most severe impairment of oxygen delivery to the tissues? a. Anemia : Hb of 95 b. Hypoxia:arterial PO2 of 60, PH 7.40 c. Constrictive pericarditis: cardiac output fixed at 1.0L/min d. Hemoglobinopathy: sickle cell trait e. Polycytemia: HB of 170 88-A 2 y old develops blister-like lesions clustered under the axilla.Some lesions are umbilicated.The best management option would be: a. Apply topical tretinoin b. Await natural involution c. Curette the lesions d. Have parents pick off the session each day e. Freexe lesions with liquid nitrogen 89-Which of the following is the most common gynecologic malignancy in Canada? a. Endometrial carcinoma b. Ovarian cancer c. Vulvar cancer d. Vaginal cancer e. Fallopian tube cancer

90-A 54 y old male has become forgetful, preoccupied, withdrawn,suspicious and disheveled.His physical examination was normal.The patient has been with his company for 22 years and was considered an excellent employee.Which of the following is the most likely diagnosis? a. Multi infarct dementia b. Hypothyroidism c. Schizophrenia d. Alcoholism e. Alzheimer`s 91-A 70 y old woman complains of right sided hearing loss.Neurological examination demonstrates right sensory neural hearing loss, a reduced right corneal reflex and mild ataxia on tandem gait.The rest of the examination is normal.What is the most likely diagnosis? a. Right acoustic neuroma b. Brain stem glioma extending from the mid brain to the lower pons c. Giant basilar termination aneurysm d. Meningioma of the anterior foreamen magnum e. Chordoma of the right cavernous sinus and temporal bone 92-Which of the following is the most common long term complication of shunting procedures for portal hypertension? a. Increased incidence of ulcer disease b. Development of hemochromatosis c. Production of a malabsorptive state d. Increased incidence of hepatic encephalopathy e. Increased incidence of diabetes 93-A 53 y old diabetic man complains of hand numbeness and weakness.On exam, there is wasting,weakness, and fasciculations of the interosseous muscles of the right hand only with sparing of abductor pollicis brevis and normal reflexes. His symptoms are reproduced by striking over the medial surface of the elbow. Which of the following is the most likely cause? a. Diabetic Polyneuropathy b. Ulnar nerve entrapment c. Carpal tunnel syndrome d. Syringomyelia e. Apical lung tumour (Pancoast) 94-A 7 year old has a persistent night-time cough for 1 year.The most likely cause of the patients cough is: a. Foreign body aspiration b. Immotile cilia syndrome c. Psychogenic cough d. Reactive airway disease e. Tuberculosis 95-Which of the following is the best treatment for agoraphobia with panic? a. lorazepan 1mg TID b. Fluoxetine 20mg OD c. Fluphenazine 20mg OD

d. Cognitive therapy e. Lithium 300mg QID 96-Exercise induced asthma is most likely to occur when an adolescent: a. Dances at school prom during the winter b. Goes horseback riding in the mountains during summer c. Goes swimming d. Plays soccer on a day with a high smog level e. Plays softball on a humid summer day 97-A 51 y old woman who was previously well complains of excruciating headache which started after sexual intercourse 2 days ago.The headache was primarily initially frontal but it has become occipital in nature.During the 2 days she has had some photophobia.Which of the following is the single most likely diagnosis? a. A resolving intracerebral hematoma b. Subarachnoid hemorrhage due to a ruptured aneurysm c. A cerebral infarction due to a carotid embolus d. A carotid dissection with cerebral infarction e. Cerebral abscess of hematogenous origin 98-In a randomized trial,the randomization process fails to produce equivalent distribution of all measured variables in both the treatment and control groups.Which aspect of the research validity could be threatened by this failure? a. Internal validity b. External validity c. Construct validity d. Face validity e. None of the above 99-A 5 y old has lumps in her neck for 6 weeks.One month ago she was treated with Amoxicilin-Clavulinate for 1 week. The lumps did not become smaller nor fluctuant.The first step in evaluating this patient is to obtain: a. CBC and differential b. Detailed history that evaluates contact with animals c. MRI scan of the neck d. CT scan of the neck with IV contrast e. Material aspiration of the largest lump 100-Four days after undergoing an Achilles tendon repair, a 28 y old male has pain in the calf. Physical examination reveals tenderness and swelling in the calf.Which of the following would be the next step in investigation? a. Monitor compartment pressures b. Venous Doppler flow studies c. Venography d. Complete blood count and differential e. A ventilation-perfusion scan Answer key (31 to 100) 31- D

32-B 33-D 34-NOT SHOWN 35-D 36-C 37-E 38-D 39-D 40-NOT SHOWN 41-D 42-A 43-B 44-E 45-B 46-A 47-E 48-NOT SHOWN 49-NOT SHOWN 50-D 51-C 52-D 53-B 54-C 55-E 56-C 57-B 58-D 59-B 60-A 61-B 62-E 63-D 64-B 65-C 66-E 67-B 68-D 69-C 70-D 71-E 72-B 73-C 74-D 75-B 76-C 77-D

78-B 79-E,D 80-B 81-D 82-C 83-B 84-A 85-D 86-B 87-C 88-B 89-A 90-E 91-A 92-D 93-B 94-D 95-B 96-D 97-B 98-A 99-B 100-C 101- Which of the following is the most reliable method of detecting a retained succenturiate lobe? a. Inspection of the fetal side of the placenta b. Inspection of the maternal side of the placenta c. Inspection of the Umbilical cord d. Abdominal palpation of the uterus e. Vaginal visualization of the lower uterine segment 102-A sexually active woman presents with green foul smelling discharge,and clue cells are seen under the microscope.The best treatment for this patient would be: a. Oral ampicilin b. Oral septra c. Oral tetracycline d. Oral metronidazole e. Vaginal Metronidazole 103-Which of the following exemplifies a type of thought that is symptomatic of schizophrenia? a. The television sending subliminal messages b. The foul odor emanating from the ventilation system (in order to poison the individual) c. Thoughts that jump from topic to topic d. The fear of heights e. God is talking to the individual

104-A 3 y old has unilateral purulent rhinorrhea for the past 3 days.The most likely diagnosis would be: a. Cartilagenous septal deviation b. Choanal atresia c. Encephalocele d. Foreign body e. Juvenile angiofibroma 105- A 12 weeks gestational age, the uterine fundus should be palpable at: a. The umbilicus b. Just above the pubic symphysis c. Midway between the symphysis and the umbilicus d. Midway between the umbilicus and xiphoid e. Within the pelvis 106-You are asked to see a patient with a pulsatile abdominal mass.Which of the following would you obtain for diagnosis? a. Arteriogram b. CT scan c. Abdominal Ultrasound d. MRI e. Duplex scan 107-The most common cause of a hemothorax after blunt trauma is a: a. Lung injury b. Ruptured pulmonary artery c. Ruptured thoracic aorta d. Laceration of intercostals vessels e. Ruptured diafragma 108-Which of the following is NOT a characteristic of delirium? a. Anxiety b. Disorientation c. Hallucinations d. Labile moods e. Neologism 109- A 56 y old male presents to your office complaining of back pain,generalized fatigue,weight loss and loss of appetite.From the general physical examination,you identify some nodularity of the prostate.A serum PSA was obtained and the value was 540.The patient was referred to a urologist and a biopsy was performed , showing a Gleason score 9/10 adenocarcinoma.A bone scan was done demonstrating wide-spread metastatic disease. The most appropriate therapy for this patient at the present time would be: a. Radical prostatectomy b. Radiotherapy c. Strontium radiotherapy d. Hormonal therapy in the form of total androgen blockade e. IV chemotherapy 110-A 35 y old woman develops extremely pruritic, purple, polygonal papules on her wrists.In association with these skin findings,she is likely to have:

a. Herpes labial on the lips b. B. Candidiasis on the vulva c. Lacy white patches on the buccal mucosa d. Ulcerations on the conjuctiva e. Anal fissures 111-A 75 y old lady, with a history of atrial fibrillation of 10 years durations and a 2 year history of carcinoma of the ovary controlled with chemotherapy,suddenly reverts to sinus rhythm.Subsequently,she complains of severe abdominal pain but nothing is found on physical examination.The most likely cause is: a. A perforated duodenal ulcer b. An embolus to the superior mesenteric artery c. Acute pancreatitis d. An hysterical reaction e. None of the above 112-Which of the following features distinguishes optic neuritis from papilledema? a. Visual acuity is preserved with papilledema b. Optic neuritis enlarges the blind spot c. The disc appears normal with papilledema d. A relative afferent papillary defect is seen with papilledema e. Venous pulsations are absent with optic neuritis 113-The major cause of perinatal death with twin gestation is: a. Asphyxia during labour b. Cord prolapse or entanglement c. Twin-twin transfusion d. Prematurity e. Abruptio placenta 114-A 3 day old term infant has episodes of breathing for 10-15 seconds followed by apnea for 5-10 seconds. There are no changes in skin color or heart rate.Which of the following is the most appropriate step in management? a. Administer caffeine b. Administes theophylline c. Apply continuous positive airway pressure d. Provide supplemental oxygen e. Reassure the parents 115-Which of the foloowing is a correct estatement about hemolytic anemia? a. The serum bilirubin is a sensitive indicator of hemolysis b. In cold agglutinin disease, IgM is detected on the red cell surface c. In warm auto-immune hemolytic anemia, the indirect Coombs`s test is positive d. Oxidant stress hemolysis is characterized by bite cells e. Approximately 5% of patients with CML will have hemolytic anemia 116- A 24 y old G1 P0 is seen in the emergency department> Her LMP was 8 weeks ago. She is experiencing lower abdominal cramping and heavy vaginal bleeding with clots. Esxamination reveals a soft abdomen with mild lower abdominal tenderness. On pelvic exam,the vagina is filled with blood and clots. The cervical os is opened and tissue is protrunding. The uterus is enlarged to a 6 weeks size. Which of the following is the most likely diagnosis?

a. Ectopic pregnancy b. Threatened abortion c. Degenerating fibroid d. Placenta previa e. Incomplete abortion 117-The major reason for advocating esophageal hiatal hernia repair through the abdomen is: a. A better anti-reflux repair can be obtained with an abdominal approach b. Gastric resection and piloroplasty are easier than through the chest c. Post-operative chest pain can be avoided d. The colon can be better prepared for interposition e. Associated abdominal disease is not uncommom and can be better dealt with in the abdomen 118-Concerning the use of pneumococcal vaccine for a 6 month old child with sickle cell disease,which of the following statements is correct? a. Vaccine may be used instead of daily penicillin prophylaxis b. Immunization at 12 months and annually thereafter prevents mild as well as serious disease c. Immunization at any time does not help prevent serious pneumococcal disease d. Immunization given at 6 monmths of age will prevent serious pneumoccocal disease e. Child should receive prompt medical attention for any fever,regardless of immunization status 119-A 25 y old surgical nurse is concerned he is losing his mind. For the past 6 months he`s been preoccupied with contamination on his ward.He has been avoiding touching patients,door knobs ,etc., and has been washing his hands excessively. Which of the following treatments is most likely to decrease his preoccupation and his hand washing? a. Fluoxetine b. Lorazepan c. Perphenazine d. Insight-oriented psychotherapy e. Nifedipine 120- Eletroconvulsive therapy is useful in the treatment of : a. Depersonalization syndromes b. Mania c. Conversion disorder d. Chronic schizophrenia e. Dysthymic disorder 121- Delivery of the aftercoming head in a breech delivery is best facilitated by which of the following? a. Traction on the neck b. Mann`s forceps c. Suprapubic pressure d. Piper`s forceps e. Barton`s forceps 122- The patient most likely to develop adenomatous hyperplasia is:

a. A 24 y old on low dose oral contraceptives b. An obese female with a history of oligoovulation and infertility c. A post-menopausal black female with multiple fibroids d. A 30 y old G3 P3 female with dysfunctional uterine bleeding e. A 23 y old student with anorexia nervosa 123- A 16 y old cuts his hand on a rusty farm implement.His most recent immunization was at school entry.What decision would you take about immunization for him now? a. Immunize using adult strength diphteria and tetanus toxoid (dT) b. Immunize using dT vaccine and tetanus immune globulin c. Withhold immunization against tetanus because he`s still adequately immunized d. Withhold immunization against tetatnus pending determination of tetanus antibody titre e. Withhold immunization against tetanus until possibility of immunosuppression is excluded 124-Which organ is the most injured after blunt trauma? a. Colon b. Stomach c. Liver d. Spleen e. Pancreas 125-A 50 y old woman notes the sonset of generalized itching 3 weeks after starting estrogen therapy. On physical examination, she has mild hepatomegaly,jaundice and dry eye. Wjhich of the following is the mnost specific diagnostic therapy test? a. Antinuclear antibodies b. Antimitochondrial antibodies c. Anticentromere antibodies d. Antineutrophil antibodies e. Antismooth muscle antibodies 126- A 47 y old male comes to the ER complaining of difficulty breathing.He says his breathing has been noisy over then past 2 to 3 days but now he feels that he can`t get enough air.On examination he is obviously in respiratory distress.He has inspiratory stridor. He is using his accessory muscles of respiration and his respiratory rate is 40. The next step in approprate management for this patient would require: a. Oxygen 5 l/min NP b. Ventolin inhalation c. Take him to the O perating room to secure the aieway d. Aminophylline drip e. Chest X-ray 127- A 30 y old Scottish woman has bilateral leg weakness and urinary retention.Which of the following tests is most likely to establish the diagnosis? a. CT scan of the head b. Lumbar puncture c. Radiographs of the thoraco-lumbar spine d. Evoked potentials 128-A 37 y old man has 12 h of progressive epigastric pain complicated by nausea and vomiting.He has a 10 year history of alcohol abuse.Epigastric tenderness is present on

examination.The serum amylase level is 1050micro/L (normal less than 150micro/L). A plain film of the abdomen showed several small bowel loops with air fluid levels. The most likely diagnosis is: a. Acute gastritis b. Acute pancreatitis c. Perforated Peptic Ulcer d. Acute cholecystitis e. Partial small bowel obstruction 129- The principal lethal component in producing death from strangulated ,gangrenous bowel is: a. Electrolyte deficiency b. Haemolysis c. Fluid loss d. Hyperkalemia e. Bacterial toxins 130-Which of the following is LEAST effective in the treatment of schizophrenia: a. Fluspirilene b. Phenelzine c. Thiothixene d. Risperidone e. Thioridazine 131- A patient presents to your office with a chronic venous stasis ulcer above and behind the right malleolus.Your immediate management would include: a. Coumadin b. Schedule patient for perforator ligation c. Order a gradient compression stock and follow-up d. Discuss the potentiasl of amputation e. None of above 132-The most effective initial treatment of a patient with systemic necrotizing vasculitis is: a. Azathioprine 2.0mg/kg/day b. Intravenous pulse Cyclophosphamide 0.5g/m2 for 2 doses each month c. Intravenous gammaglobulin d. Plasmapheresis e. High-dose systemic corticosteroid therapy 133-A healthy 7 year old girl has periodic vomiting for 2 years;each episode lasts a few days.She has been hospitalized frequently for treatment of dehydratation.Physical exam and upper GI series are noermal.What is themost likely diagnosis? a. Duplication cysts of the stomach b. Food intolerance c. Cyclic vomiting d. Intracranial lesion e. Metabolic disorder 134-Hemorrhagic shock is LEAST likely to be due to which of the following? a. Ruptured spleen b. Massive hemothorax

c. Unstable pelvic fracture d. Acute epidural hematoma e. Bilateral open femoral fractures 135-A young male was in a motorcycle crash and presents to the ER unresponsive to deep pain and with a left pupil that is dilated and unreactive. The right pupil is normal.The most appropriate initial treatment is: a. Mannitol b. Insertion of an Intravenous catheter c. Tetanus Toxoid d. CT scan of the head e. Endotracheal intubation 136-A 42 y old female presents to your office with back leg pain and difficulties urinating.On examination you find decreased sensation over the buttocks,normal motor power and absent ankle jerks bilaterally.What is the most appropriate next step in management of this patient? a. Bedrest and follow-up in 2 weeks b. Physiotherapy and back education c. Book a CT-myelogram of the lumbar spine d. Urgent discectomy e. Obtain plain radiographs of the L-spine 137- A 2 y old with fever is seen in the hospital following a generalized seizure.He also has foul smelling,blood tinged diarrhea.The culture of the stool is most likely to grow: a. Campylobacter species b. E.coli c. Entamoeba histolytica d. Salmonella species e. Shigella species 138-In which way does behaviour therapy differ from insight-oriented psychotherapy? a. The use of empathy b. The emphasis on understanding the origins of maladaptative behaviour. c. The adjunctive use of medication d. The formation of a therapeutic alliance e. Self revelation on the part of the therapist 139-The most common clinical picture seen in blind lop syndrome after side-to-side intestinal anastomosis is: a. Mycrocytic anemia b. Watery diarrhea c. Bulky stools d. Megaloblastic anemia e. Cramping abdominal pain 140-A 23 y old male was involved in a snowmobile accident and sustained a closed,displaced humeral shaft fracture. His neurovascular status is normal.You realign the fracture and place him in a splint.Prior to discharge he states ha has numbness over the dorsum of his hand and he is unable to extend his metacarpophalangeal joints.Which of the following would you suspect to be the cause? a. Ulnar nerve was damaged at the time of the fracture

b. Radial nerve was damaged at the time of the fracture c. Radial nerve is trapped in the fracture site d. Ulnar nerve is compressed by hematoma e. Median nerve is trapped in fracture site 141-A 34 yold labourer presents to you with severe back and right leg pain after lifting some heavy pieces of wood.The pain radiates from his low back down to the posterior lateral aspect of the leg into the lateral foot.The pain is associated with a sensation of swelling in the lateral aspect of his right foot.Staight leg raising is limited to 40 degrees.Lasegue sign is + but the Bowsting sign is -. What additional signs would you expect to find in this patient? a. hyper-reflexia with clonus on both legs but more so on the right. b. A suspended disassociated sensory loss from T12 to S1 c. Wasting of the right quadriceps muscle with an absent right knee reflex d. An absent right ankle reflex e. Weakness of ankle dorsiflexion and reduced sensation over the right halux 142-In a Q wave myocardial infarctionwhich of the following is the primary pathophysiologic mechanism? a. Vasospasm b. Fissuring on an atheroma without thrombus c. Occlusive thrombus on a major plaque rupture d. Fixed atheroma e. Subocclusive thrombus on a plaque fissure 143-A 23 y old man is involved in a car crash after drinking and driving.At the ER, your initial examination reveals his blood pressure to be 140/80,heart rate of 74 with no evidence of chest or extremities injury.He is opening his eyes to pain and mumbling incomprehensible sounds.When you apply a painful stimulus to his supraorbital nerve, he reaches up and grabs your hand and pulls it away. What score would you give him on the Glasgow Coma scale? a. 12 b. 65 c. 3 d. 9 e. 15 144-You have seen a 28 y old woman who presents with infertility and secondary amenorrhea with a serum prolactin of 60mcg/L and a 3 mm microadenoma on CT scan.Your management would be to recommend: a. Bromocryptine b. Radiation therapy c. Estrogen replacement d. Clomiphene e. Human menopausal gonadotrophin 145-Which of the following maneuvers has the highest sensitivity and specificity for the diagnosis of a testicular torsion? a. Physical examination b. Doppler c. Ultrasound

d. Testicular scan e. Incision of scrotum and open exploration 146-Polyhydramnios (excessive amniotic fluid) is associated with which one of the following fetal or maternal conditions? a. Potters Syndrome (renal agenesis) b. Anencephaly c. Placental insufficiency d. Posterior urethral valves e. Prune belly syndrome 147-Which of the following findings is most consistent with a diagnosis of learning disability? a. Normal score on Intelligence test b. Normal score on Connors Rating Scales for rating hyperactivity c. Abnormal findings on Audiometry d. Abnormal findings on EEG e. Abnormal findings on neurologic examination 148-A male patient is recovering from an episode of major depression. Five years ago he had a similar episode which resolved spontaneously after 8 months.For this patient,which of the following statements is correct? a. He will likely have episodes less frequently in the future b. There is a 40% chance he will develop bipolar mood disorder in the future c. There is a 15% chance he will eventually kill himself d. He is at an increased risk for developing Alzeheimer`s disease e. His antidepressant medication should be discontinued shortly after his symptoms remit to prevent a switch into mania 149-A 24 y old female G1 P0 is admitted to hospital at 34 weeks gestation.The patient describes passing large amounts of clear fluid per vagina for 36 h.Examination reveals the following: Fetal HR 185 beats/min.Lab incvestigations reveal: Hgb 120,WBC 19.0, +ve Nitrazine test of vaginal fluid,+ve Ferning of vaginal fluid.Which of the following is the most likely diagnosis? a. Premature rupture of membranes b. Premature ruptured membranes and Chorioamnionitis c. Premature labour d. Acute Pyelonephritis e. Acute vaginitis of pregnancy 150-The difference between a common point source outbreak and a propagated outbreak of illness is that: a. All cases in a common point source outbreak occur within one incubation period of the exposure b. The attack rates in propagated outbreaks are higher c. Person-to-person transmission is a feature of common source outbreaks d. Case fatality rates in common source outbreaks are generally higher e. The source of infection in propagated outbreaks is more easily contained than in common source outbreaks 151-Which of the following is essential in making a diagnosis of schizophrenia? a. Hallucinations

b. Delusions c. Deterioration of functioning d. Disorganized thinking e. Onset before age 45 152-The subendothelium is the most vulnerable segment of the heart from an ischemic standpoint.The major reason for this is: a. The highest O2 utilization is in the endocardium b. Coronary flow to the subendocardium occurs almost completely during diastole whereas other regions receive some flow during systole as well c. The subendocardium has a diminished aerobic capacity d. There is less potential for collateralization to the subendocardium e. The ratio of capillary to myocyte is less in the subendocardium than in other regions 153-A 59 y old woman presents with a progressive history of confusion for over 1 month.On examination,she is found to have an expressive dysphasia and mild right arm weakness. Where is the lesion most likely to be? a. Right parietal lobe b. Left parietal lobe c. Right frontal lobe d. Left frontal lobe e. Cannot be explained by one single lesion 154-A 34 y old lady presents with progressive dysphagia to both solids and liquids.Heartburn is not a feature of her history. A barium swallow reveals a dilated esophagus with a hold up of barium and a rapid taper at the cardia (Bird`s beak deformity).The most likely diagnosis is: a. Squamous cell carcinoma of the esophagus b. Adenocarcinoma extending into the esophagus c. Achalasia d. Scleroderma e. Peptic stricture 155-What is the most appropriate first-line form of therapy for a uric acid calculus? a. ESWL b. Percutaneous nephrolithotomy c. Uretoscopy d. Alcalinization of urine e. Expectant therapy 156-A 5 y old boy has a seizure that began 45 min ago.Initial treatment includes oxygen and anticonvulsant.History and physical exam reveals no precipitant cause.Which of the following tests would be the most urgent to obtain? a. Toxicology screen b. EEG c. CT scan of the head d. Serum glucose and electrolytes e. Serum ammonia and lactic acid 157-A 26 y old student has been well controlled on lithium for 5 years.His last hospitalization was for a severe psychotic depression.He is now complaining of a 6 week

history of dysphoria,lack of drive and sleepiness.Your most appropriate initial intervention should be: a. Stop the lithium b. Do thyroid function tests c. Start an antidepressant d. Start an antidepressant and a neuroleptic e. Check his hemoglobin 158-Which of the following is the most appropriate test for monitoring a hypothyroid patient`s thyroid status while on Thyroxine? a. Free T4 b. Free T3 c. TSH d. FT4-I e. Radioactive iodine uptake 159-A 73 y old man with CHF but no previous GI symptoms suddenly develops severe abdominal pain. Three h later he passes dark red bloody stool. On a plain film of the abdomen,colonic gas outlines a coarsely nodular colonic mucosa (thumb printing) in the region of the splenic flexure and descending colon.There is no free air in the abdomen.Which of the following should be the next step in the initial management of this patient? a. Mesenteric angiography b. Immediate surgery c. Gastroscopy d. Steroid therapy e. Continued close observation 160-The LEAST compelling evidence of a causal relationship in a clinical study is: a. Temporal priority b. Evidence from a cohort study c. Evidence from a case-control study d. Biologic plausibility e. Analogy to other similar situations 161-When interpreting a Tuberculin skin test, one measures: a. The total area of redness b. The radius of the induration c. The maximal induration of the transverse diameter d. The induration divided by the redness e. None of the above 162-What is the most appropriate therapy for a muscle invasive transitional cell carcinoma of the bladder associated with palpable mass on pelvic exam? a. Radical radiotherapy b. Radical cystectomy and urinary diversion c. Transurethral resection of the bladder tumour d. Intravesical BCG e. None of the above

163-A 1100Gm. Infant born at 26 weeks gestation develops respiratory distress syndrome and requires supplemental 02 until 4 months of age.A chest X-ray then shows hyperinflation of both lungs and cystic changes in the bases.The most likely diagnosis is: a. Bronchopulmonary dysplasia b. Cor pulmonale c. Persistent pulmonary hypertension d. Pneumonia e. Cystic adenomatoid malformation of the lung 164-A 72 y old female presents to your office with a recent episode of transient left hand weakness lasting approximately 10 min.In addition, a few months previously she noted a brief episode of transient blindness in her right eye.What problem would you first suspect? a. Left atrial thrombus b. Diffuse intracranial cerebrovascular disease c. Left carotid stenosis d. Right carotid stenosis e. None of the above 165-A 40 y old male sustained a displaced subcapital hip fracture after a fall while rollerblading.This was treated with multiple pin fixation.He presents to your clinic 5 months after the operation with increasing hip pain.The x-rays demonstrate narrowing of the joint space and subchondral sclerosis.What is the most likely diagnosis? a. Nonunion b. Osteonecrosis c. Pins backing out d. Malunion e. Septic arthritis Note: Questions 166-200 ask for the EXCEPTION 166-A blood tinged vaginal discharge in a 4 y old child may be caused by each of the following, EXCEPT: a. Sand b. Gonococcus c. Enterobius vermicularis d. Sarcoma botrioides e. Nabothian follicle 167-Known risk factors for malignat melanoma include each of the following, except: a. Multiple seborrhoeic keratoses b. Ultraviolet B radiation c. Multiple dysplastic nevi (atypical moles) d. Family history of malignant melanoma e. History of repeated sunburn 168-In a 3 y old child each of the following would be described as a normal behavioure EXCEPT: a. Masturbation b. Thumb sucking

c. Attachment to an inanimate object a such as a stuffed animal d. Inconsolable distress when separated from mother e. Can run easily and negotiate stairs alone with alternate feet 169-Each of the following is a correct statement about Chronic Lymphocytic Leukemia, EXCEPT: a. CLL is the most prevalent Leukemia in North America b. Approximately 50% of patients will have a para-proteinb (M-component) at some point in their clinical course c. Fludarabine is a purine analogue chemotherapeutic agent used in CLL d. Infections are the most common cause of death in patients with CLL e. Autoimmune thrombocytopenia autoimmune hemolytic anemia are more common in CLL patients than in normal population 170-Nutritional guidelines for patients with ndiabetes include each of the following EXCEPT: a. A diet low in satured fat b. A diet high in carbohydrate c. A diet high in soluble fibre d. Strict avoidance of all sucrose e. A eucaloric diet if the patient is of normal weight 171-Each of the following is a cardinal feature of an acute suppurative tenosynovitis of a digit, EXCEPT: a. Digit held in flexed position b. Pain on compression of the PIP joint (positive grind test) c. Fusiform swelling of the digit d. Pain on passive extension e. Tenderness to palpation over the flexor tendon sheath 172-Presumptive evidence of ovulation includes each of the following,EXCEPT: a. Mittelschmerz b. Elevated serum progesterone c. Elevated basal body temperature d. Elevated prolactin e. Thin, watery discharge 173-Important advantages to conducting a Case-Control study rather than a Cohort study includes each of the following, EXCEPT: a. Less Expensive b. Requires only a small numbers of subjects c. Will yield estimates of incidence rates of disease d. Suitable for rare diseases e. Relative speed 174-Each of the following statements about delusional disorders is correct, EXCEPT: a. They are less common than schizophrenia b. The patient often functions well at work c. It is unlikely the patient will go on to develop schizophrenia d. The delusions respond well to neuroleptic medication e. Delusions are never bizarre

175-Occupational lung cancer is known to be caused by exposure to each of the following agents, EXCEPT: a. Arsenic b. Asbestos c. Aromatic amines d. Chromium e. Hydrocarbons 176-Management of a flail chest may include each of the following EXCEPT: a. Supplemental O2 b. Endotracheal intubation c. Tube thoracostomy d. Epidural analgenia e. Strapping of the chest 177-A tuberculin skin test is read at 72h as 5mm.This is negative in each of the following, EXCEPT: a. Apical scarring on chest x-ray b. HIV + c. Previously normal Tuberculin test (within 2 years) d. Patient on cortico-steroids e. Previously documented tuberculosis 178-For investigating a patient with elevated serum prolactin level, each of the following may be useful EXCEPT: a. A serum TSH b. A serum HCG c. A MRI of the sella turcica d. A mammogram e. Obtaining a history of psychiatric disease 179-Impaired coronary flow reserve is associated with each of the following conditions EXCEPT: a. Severe Aortic stenosis b. Severe systemic hypertension with left ventricular hypertrophy c. Severe mitral stenosis in the presence of atrial fibrillation d. A totally occluded coronary artery but with excellent collateral supply from the contralateral coronary artery e. An isolated 30% diameter stenosis of a coronary artery 180-Acute diffuse proliferative glomerulonephritis is usually accompanied by each of the following, EXCEPT: a. Red blood cell casts on microscopic urinalysis b. Proteinuria c. Pigmented casts on microscopic urinalysis d. Blood clots in the urine e. None of the above 181-The health effects of non-ionizing radiation include each of the following EXCEPT: a. Photokeratitis b. Loss of hair c. Cataracts

d. Skin cancer e. Actinic changes 182-In the treatment of a repetitive suicidal university student who has a personality disorder , each of the following would be recommended ,EXCEPT: a. Brief hospitalization b. Pharmacotherapy c. Psychoanalysis d. Mobilization of social supports e. Behaviour therapy 183-Severe hyperkalemia is frequently seen in acute oliguric renal failure.Each of the following ECG changes is helpful in confirming the diagnosis of progressive hyperkalemia EXCEPT: a. Peaking of T waves b. ST segment depression c. Widening of PR interval d. Widening of QRS interval e. Peaking of P waves 184-Concerning childhood psychiatric disorder, each of the following statements is correct, EXCEPT: a. Child with mild mental retardation are identifiable at age 2-3 years b. Children with developmental reading disorder have normal intelligence c. The prevalence of autism in siblings of autistic children is 50 times that in the general population d. The diagnosis of encopresis can be made when a child is 4 years old e. Children with gender identity disorder rarely go on to develop transsexualism in adult life 185-Each of the following is a primary preventive strategy in the prevention of occupational illness EXCEPT: a. Engineering control b. Surveillance c. Education d. Protection equipment e. Hygienic practice 186-Each of the following factors is associated with endometrial cancer EXCEPT: a. Age 75 b. Diabetes c. Hypertension d. Obesity e. Multiparity 187-In the case of a 50 y old woman`s first seizure, each of the following is a likely cause EXCEPT: a. Brain tumour b. Alcohol abuse c. Prior head injuries d. Multiple Sclerosis e. Subdural hematoma

188-In the use of methylphenidate in children, side effects might include each of the following EXCEPT: a. Sleeplessness b. Weight gain c. Itchy skin d. Hyperactivity e. Growth retardation 189-OFFICIALLY CANCELLED 190- Not shown (requires Figure) 191-each of the following treatment modalities has been shown to be effective in the management of Chron`s Disease EXCEPT: a. Steroids b. Cyclosporin c. Metronidazole d. 6-Mercaptopurine e. Methotrexate 192-Each of the following infectuos organisms is a toxin producer EXCEPT: a. Diphtheria b. Botulism c. Tetanus d. Staphylococcus e. Pneumococcus 193-Treatment of hypertension caused by bilateral renal artery stenosis should include each of the following EXCEPT: a. Weight reduction if obese b. Drug therapy with angiotension-converting enzyme inhibitors c. Percutaneous transluminal renal angioplasty d. Surgical renal revascularization e. Sodium restriction 194-Each of the following diseases is associated with hepatic failure EXCEPT: a. Benign recurrent cholestasis of pregnancy b. Fatty liver of pregnancy c. Toxemia of pregnancy d. Viral hepatitis during pregnancy e. None of the above 195-Symptoms of CHF includes each of the following EXCEPT: a. Paroxysmal nocturnal dyspnea b. Swelling of the ankles c. Upper right quadrant pain d. Decreased exertional capacity e. Hoarseness 196-The presence of jaundice is explained by each of the following EXCEPT: a. Alcoholic Hepatitis b. T3N1 Adenocarcinoma of the ampulla of Vater c. Choledochal cyst d. Malignant stricture of the left hepatic duct

e. Dubin-Johnson syndrome 197-Complications of genital herpes virus infections include each of the following EXCEPT: a. urinary retention b. Pelvic Inflammatory disease c. Secondary bacterial skin infection d. Transverse myelitis e. Aseptic meningitis 198-Absolute contraindications of birth control pill include each of the following EXCEPT: a. Smoking b. Pregnancy c. Active liver disease d. Diagnosed vaginal bleeding e. Previous thromboembolic disease 199-The anion gap is increased in Metabolic Acidosis associated with each of the following EXCEPT: a. Diabetic ketoacidosis b. Renal tubular acidosis c. Acute Tubular necrosis d. Ethylene glycol intoxication e. Lactic acidosis 200-Concerning an adolescent who is frequently truant,sexually exploitive, lies, steals and abuses drugs,each of the following statements is correct EXCEPT: a. there is a good chance he has a reading disability b. He likely has no remorse or guilt c. He may have soft neurologic signs d. He is likely isolated from his peers e. He should receive intensive insight-oriented individual psychotherapy

ANSWER KEY: 101 to 200 101-A 102-D 103-A 104-D 105-A 106-C 107-D 108-E 109-D 110-C 111-B 112-A 113-D

114-E 115-D,B 116-E 117-C 118-E 119-A 120-B 121-D 122-B 123-A,B 124-D 125-B 126-C 127-D 128-B 129-E 130-B 131-C 132-E 133-C 134-D 135-E 136-D 137-E 138-B 139-D 140-C 141-D 142-C 143-D 144-A 145-E 146-B 147-A 148-C 149-B 150-A 151-C 152-B 153-D 154-C 155-D 156-D 157-B 158-C 159-A

160-E 161-C 162-B 163-A 164-D 165-B 166-E 167-A 168-D 169-B 170-D 171-B 172-D 173-C 174-D 175 -C 176-E 177-B 178-D 179-E 180-D 181-B 182-C 183-E 184-A 185-B 186-E 187-D 188-B 189-DELETED 190-D 191-B 192-E 193-B 194-A 195-E 196-D 197-B 198-A 199-B 200-E

Вам также может понравиться